You are on page 1of 63

2017-18 100 &

op kers
Class 12 T
By E ran culty
-JE Fa r
IIT enior emie .
S fP r es
o titut
Ins

MATHEMATICS
FOR JEE MAIN & ADVANCED
SECOND
EDITION

Exhaustive Theory
(Now Revised)

Formula Sheet
9000+ Problems
based on latest JEE pattern

2500 + 1000 (New) Problems


of previous 35 years of
AIEEE (JEE Main) and IIT-JEE (JEE Adv)

5000+Illustrations and Solved Examples


Detailed Solutions
of all problems available

Topic Covered Plancess Concepts


Tips & Tricks, Facts, Notes, Misconceptions,
Area Under the Curve and Key Take Aways, Problem Solving Tactics
Linear Programming
PlancEssential
Questions recommended for revision
25. AREA UNDER THE
CURVE AND LINEAR
PROGRAMMING

AREA UNDER THE CURVE


1. INTRODUCTION
In the previous chapters we have studied the process of integration and its physical interpretation. The most
important application of integration is finding the area under a curve. In this topic we will discuss different curves
and the area bounded by some simple plane curves taken together. In order to find the area, we need to know the
basics of plotting a curve and then use integration with appropriate limits to get the answer. The process of finding
area of some plane region is called Quadrature.

2. CURVE TRACING
Let us now discuss the basics of curve tracing. Curve tracing is a technique which provides a rough idea about
the nature and shape of a plane curve. Different techniques are used in order to understand the nature of the
curve, but there is no fixed rule which provides all the information to draw the graph of a given function (say f(x)).
Sometimes it is also very difficult to draw the exact curve of the given function. However, the following steps can
be helpful in trying to understand the nature and the shape of the curve.
Step 1: Check whether the origin lies on the given curve. Also check for other points lying on the curve by putting
some values.
Step 2: Check whether the curve is increasing or decreasing by finding the derivative of the function. Also check
for the boundary points of the curve.
Step 3: Check whether the curve f(x , y) = 0 is symmetric about
(a) X-axis: If the equation remains same on replacing y by –y i.e. f(x , y) = f(x , –y), or, if all the powers of “y” are even,
then the graph is symmetric about the X-axis.
(b) Y-axis: If the equation remains same on replacing x by –x i.e. f(x , y) = f(–x , y), or, if all the powers of “x” are even,
then the graph is symmetric about the Y-axis.
(c) Origin: If f(–x, –y) = –f(x, y), then the graph is symmetric about the Origin.
For example, the curve given by x2 = y+2 is symmetrical about y-axis, y2 = x+2 is symmetrical about x-axis and the
curve y = x3 is symmetrical about the origin.
Step 4: Find out the points of intersection of the curve with the x-axis and y-axis by substituting y = 0 and x = 0
respectively.
x2 y 2
For example, the curve + 1 intersects the axes at points (± 3, 0) and (0, ±2).
=
9 4
2 5 . 2 | Area Under the Cur ve and Linear Programming

Step 5: Identify the domain of the given function and the region in which the graph can be drawn.
2−x
For example, the curve xy2 = (8 – 4x) or y = 2 .
x

2−x
Therefore the value of y is defined only when ≥ 0 i.e. 0 < x ≤ 2 . Hence, the graph lies between the lines x =
0 and x = 2. x

Step 6: Check the behaviour of the graph as x → + ∞ and as x → − ∞. Find all the horizontal, vertical and oblique
asymptotes, if any.
Step 7: Determine the critical points, the intervals on which the function (f) is concave up or concave down and
the inflection points.
The information obtained from the Steps 1 to 7 are used to trace the curve.

Illustration 1: Trace the curve y2 (2a – x) = x3  (JEE MAIN)

Sol: By using curve tracing method as mentioned above.


Given curve: y2 = x3/(2a – x) ...(i)

(a) Origin: The point (0 , 0) satisfies the given equation, therefore, it passes through the origin.

(b) Symmetrical about x-axis: On replacing y by –y, the equation remains same, therefore, the given curve is
symmetrical about x-axis.

(c) Tangent at the origin: Equation of the tangent is obtained by equating the lowest degree terms to zero.
⇒ 2ay2 = 0 ⇒ y2 = 0 ⇒ y=0

(d) Asymptote parallel to y-axis: Equation of asymptote is obtained by Y


equating the coefficient of lowest degree of y to 0. The given equation can
be written as (2a – x) y2 = x3 Tangent
∴ Equation of asymptote is 2a – x = 0 or x = 2a 2a

(e) Region of absence of curve: The given equation is O X


y=O
x3
y2 (2a – x) = x3 ⇒ y2 = .
( 2a − x ) Asymptote
For x < 0 and x > 2a, RHS becomes negative, therefore the curve exists only
for 0 ≤ x < 2a .
Y’ Cissoid
Hence the graph of y2 (2a – x) = x3 is as shown in Fig. 25.1. Such a curve is Figure 25.1
known as a Cissoid.

x2 y 2
Illustration 2: Sketch the curve + 1
= (JEE MAIN)
4 9
Sol: Same as above illustration.
x2 y 2
We have, + 1
= …(i)
4 9
(a) Origin: The point (0,0) does not satisfy the equation, hence, the curve does not pass through O.

(b) Symmetry: The equation of the curve contains even powers of x and y so it is symmetric about both x and y axes.

(c) Intercepts: Putting y = 0, we get x = ± 2 i.e. the curve passes through the points (2 , 0) and (-2 , 0). Similarly, on
substituting x = 0, we get y = ± 3 i.e. the curve passes through the points (0 , 3) and (0 , -3).
M a them a ti cs | 25.3

Y
(d) Region where the curve does not exist: If x2 > 4 , y becomes imaginary. So the
(0, 3)
curve does not exist for x > 2 and x < –2. Similarly, if y 2 > 9 , x becomes imaginary. So,
the curve does not exist for y > 3 and y < –3.
X’ X
(e) Table: (-2, 0) (2, 0)

x −2 0 1 2
(0, -3)
y 0 ±3 ±2.6 0
Y’

x2 y 2 Figure 25.2
Hence the graph of + 1 is as shown in Fig. 25.2.
=
4 9

PLANCESS CONCEPTS

Using the above rules try to trace the Witch of Agnesi Y

xy2 = a2 (a – x).

(a, 0)
O X

Y’

Figure 25.3

Vaibhav Krishnan (JEE 2009 AIR 22)

3. AREA BOUNDED BY A CURVE

3.1 The Area Bounded by a Curve with X-axis y=f(x)

The area bound the curve y=f(x) with the x-axis between the ordinates
y dx
b b
x= a and x=b is given by =
Area ∫=
y dx ∫ f(x)dx
X
a a O x=a x=b
Illustration 3: Find the area bounded by the curve y = x3, x-axis and ordinates Figure 25.4: Area Bounded By a
x = 1 and x = 2.  (JEE MAIN) curve y=f(x) with x-axis

Sol: By using above formula, we can find out the area under given curve.
2 2 2
 x4  15
3
Required Area = =
∫ y dx ∫=x dx = 
 4  4
1 1  1

Illustration 4: Find the area bounded by the curve y = mx x-axis and ordinates x = 1 and x = 2. (JEE MAIN)

Sol: Same as above.


2 2 2
 mx2  m 3
Required area= ∫ y dx= ∫ mx dx=  = (4 − 1)= m
 2 
1 1  1 2 2
2 5 . 4 | Area Under the Cur ve and Linear Programming

Illustration 5: Find the area included between the parabola y2 = 4ax and its latus rectum (x = a). 
 (JEE ADVANCED)
Y
Sol: Here the curve is y 2 = 4ax , latus rectum is x = a, and the curve is symmetrical
A
about the x-axis.
2
y = 4ax

(a) The latus rectum is the line perpendicular to the axis of the parabola and x=0
passing through the focus S (a, 0). x=a

(b) The parabola is symmetrical about the x-axis. S (focus)


O X
∴ The required area AOBSA = 2 × area AOSA
a a x=0
= 2∫ y dx= 2∫ 2 ax dx  y 2= 4ax ⇒ y= 2 ax 
 
0 0

B
2 a 8 8
= 4 a.  x3/2  = a .a3/2 = a2 .
3   0 3 3
Figure 25.5

Illustration 6: Sketch the region {(x, y): 4x2 + 9y2 = 36} and find its area using integration. (JEE ADVANCED)

Sol: The given curve is an ellipse, where a = 3 and b = 2. The X and Y axis Y
divides this ellipse into four equal parts.
(0, 2)
 x2 y 2 
Region {(x, y): 4x + 9y = 36} = Region bounded by  +
2 2
1
=
 9 4 
  X’
O
X
(-3, 0) (3, 0)
Limits for the shaded area are x = 0 and x = 3.
(2, 0)
∴ The required area of the ellipse
a 3  x2 y 2  Y’
x2 y2 x2 x2

= 4= y dx 4 ∫ 2 1 −
9
dx 
 9
+
4
= 1 ⇒
4
= 1 −
9
⇒ y = 2 1 −
9


Figure 25.6
0 0  
3 3 3
8 2 2 8 x 2 9 −1  x    2 2 x 2 a2 x
= ∫ 3 − x dx = ∫  9 − x + sin    using ∫ a − x dx
= a − x2 + sin−1 
30 3 2
0 
2  3 0  2 2 a 

8 9 −1  8 9 π
= 0 + sin 1 − 0 − 0  = × × = 6π sq. units.
3 2  3 2 2

3.2 The Area Bounded by a Curve with y-Axis


Y
The area bound the curve y=f(x) with y-axis between the ordinates
y=d X
y= a and y=b is given by dx
x=f(y)

d d y=c
Area = ∫ x dy = ∫ f(y)dy O
X
c c
Figure 25.7: Area bounded by
a curve with y-axis

1
Illustration 7: Find the area bounded by the curve x2 =
y, y-axis and between the lines y = 1 and y = 4.
4
(JEE MAIN)
d d
Sol: As we know, area bounded by curve with y – axis is given by ∫ x dy = ∫ f(y)dy .
c c
M a them a ti cs | 25.5

4 4
1 2 4
Required Area = ∫ x dy = 2∫ y dy =  y 3/2  = 2 (8 − 1) =
14 sq. units
2 3  1 3 3
1 1

Illustration 8: Find the area of the region bounded by the curve y2 = 4x, y-axis and the line y = 3. (JEE MAIN)

 y 2 = 4x 
 
Sol: Same as above illustration.  y 2 
Y
2
 =x  y=3 y = 4x
 4  A
B
y =3 3
y2
Area of region
= is A ∫ x dy
= ∫ dy
y =0 0
4 X
O
3
1  y3  1  33 0  1 9
= =   −  = [9] = sq. units
4  3  4  3 3  4 4
0

9
Hence, the required area is sq. units.
4
Figure 25.8

PLANCESS CONCEPTS

There is no harm in splitting an integral into multiple components while finding area. If you have any
doubt that the integral is changing sign, split the integral at that point.
Vaibhav Gupta (JEE 2009 AIR 54)

3.3 Area of a Curve in Parametric Form


If the given curve is in parametric form say x = f(t), y = g(t), then the area bounded by the curve with x-axis is equal
b t1
to ∫ y dx = ∫ g(t)f '(t)dt = ( f(t))
 dx d=
 f '(t)dt  Where t1 and t2 are the values of t corresponding to the values
a t2

of a and b of x.

Illustration 9: Find the area bounded by the curve x = a cost, y = b sint in the first quadrant.  (JEE MAIN)

Sol: Solve it using formula of area of a curve in parametric form.

x2 y2
The given equation is the parametric equation of ellipse, on simplifying we get + 1.
=
a2 b2
a 0 π /2
 πab 
= ∫ y dx
∴ Required area= ∫ (bsint(−asint)dt) = ab ∫ sin2 t dt =  .
0 π /2 0  4 

3.4 Symmetrical Area


If the curve is symmetrical about a line or origin, then we find the area of one symmetrical portion and multiply it
by the number of symmetrical portions to get the required area.

Illustration 10: Find the area bounded by the parabola y2 = 4x and its latus rectum.  (JEE MAIN)
2 5 . 6 | Area Under the Cur ve and Linear Programming

Y
Sol: Here the given parabola is symmetrical about x – axis.
1
Hence required area = 2∫ y dx .
0
S
Since the curve is symmetrical about x-axis, O
X
(1, 0)
1 1 2 1 8
∴ The required Area = 2∫ y dx = = .  x3/2 
2∫ 4x dx 4=
0 0 3 0 3

Figure 25.9
3.5 Positive and Negative Area
The area of a plane figure is always taken to be positive. If some part of the y
area lies above x-axis and some part lies below x-axis, then the area of two
parts should be calculated separately and then add the numerical values to
get the desired area.

x=a
If the curve crosses the x-axis at c (see Fig. 25.10), then the area bounded by y = f(x)
the curve y = f(x) and the ordinates x = a and x = b, (b > a) is given by x’ x
O c
c b c b x=b
A = ∫ f(x)dx + ∫ f(x)dx ; =A ∫ f(x)dx − ∫ f(x) dx
a c a c Figure 25.10

PLANCESS CONCEPTS

To reduce confusion of using correct sign for the components, take modulus and add all the absolute
values of the components.
Vaibhav Gupta (JEE 2009 AIR 54)

Illustration 11: Find the area between the curve y = cos x and x-axis when π/4 < x < π  (JEE MAIN)
Y
Sol: Here some part of the required area lies above x-axis and some part lies below
x-axis,. Hence by using above mentioned method we can obtain required area.
/2
π /2 π
∴ Required area = ∫π / 4 cos x dx+ | ∫π /2 cos x dx | O x=/4
X
x=

= [sinx]ππ /2 π
( ) 2 2 −1
/ 4 + | [sinx]π /2 | = 1 − 1 / 2 + | 0 − 1 | =
2
Figure 25.11

Illustration 12: Using integration, find the area of the triangle ABC, whose vertices are A (4, 1), B (6, 6) and C (8, 4)
 (JEE ADVANCED)

Sol: Here by using slope point form we can obtain respective equation of line by B(6, 6)

which given triangle is made. And after that by using integration method we can
obtain required area.
C(8, 4)

5 5x A(4, 1)
Equation of line AB: y − 1= (x − 4) ⇒ =
y −9
2 2
P Q R
3 3x
Equation of line AC: y=
− 1   (x − 4) ⇒ y= −2 x=4 x=6 x=8
4
  4 Figure 25.12
M a them a ti cs | 25.7

 −2 
Equation of line BC: (y =
− 6)   (x − 6) ⇒ y = –x + 12
 2 
∴ The required area = Area of trapezium ABQP + Area of trapezium BCRQ – Area of trapezium ACRP
6 8 8
5  3 
= ∫  2 x − 9  dx + ∫ (−x + 12)dx − ∫  4 x − 2  dx
4 6 4

6 8 8
5   x2  3 2 
=  x2 − 9x  +  12x −  −  x − 2x  = 7 + 10 – 10 = 7 sq. units.
4  4  2  8
6 4

3.6 Area between Two Curves

(a) Area enclosed between two curves.


If y = f1(x) and y = f2(x) are two curves (where f1(x) > f2(x)), which intersect at two points, A (x = a) and B(x = b), then
the area enclosed by the two curves between A and B is
b b
Common area = ∫a (y1 − y 2 )dx = ∫a [f1 (x) − f2 (x)]dx
Y y=f1(x)
B

A
y=f2(x)

X
dx x=b

Figure 25.13

Illustration 13: Find the area between two curves y2 = 4ax and x2 = 4ay.  (JEE MAIN)

Sol: By using above mentioned formula of finding the area enclosed Y


between two curves, we can obtain required area.
2
x =4ay

Given, y2 = 4ax … (i)


x2 = 4ay … (ii)
Solving (i) and (ii), we get x = 4a and y = 4a.
X
4a O x=4a
4a  x2   x3/2 x3 
So required area = ∫0  4ax −  dx
= 2 a − 
 4a   3 / 2 12a 
  0

4 a 64a3 16 2
2

=
y =4ax
| 4a |3/2 − = a
3 12a 3 Figure 25.14

(b) Area enclosed by two curves intersecting at one point and the X-axis. Y

If y = f1(x) and y = f2(x) are two curves which intersect at a point P (α, β) and meet
x-axis at A (a, 0) and B (b, 0) respectively, then the area enclosed between the P(a,b)
x)

curves and x-axis is given by


f1 (
Y=

Y=
f 2(

α b
x)

Area = ∫ f1 (x)dx + ∫ f2 (x)dx O A(a,0) (a,0) B(b,0)


X
a α

Figure 25.15
2 5 . 8 | Area Under the Cur ve and Linear Programming

(c) Area bounded by two intersecting curves and lines parallel to y-axis.
The area bounded by two curves y = f(x) and y = g(x) (where a ≤ x ≤ b), when they intersect at x = c ∈ (a, b), is given
b c b
by
= A ∫ | f(x) − g(x) | dx ⇒ =
A ∫ (f(x) − g(x))dx + ∫ (g(x) − f(x))dx
a a c

Y y=f(x) y=g(x)

X’ X
O x=a x=c x=b
Y’
Figure 25.16

Illustration 14: Draw a rough sketch of the region enclosed between the circles x2 + y2 = 4 and (x – 2)2 + y2 =4. Using
method of integration, find the area of this enclosed region  (JEE ADVANCED)

Sol: By solving given equations simultaneously, we will be get intersection points of circles and then by using
integration method we can obtain required area.
The figure shown alongside is the sketch of the circles
x2 + y2 = 4  … (i)
and, (x – 2) + y = 4
2 2
… (ii)
From (i) and (ii), we have (x – 2)2 - x2 = 0 Y

⇒ (x – 2 – x)(x – 2 + x) = 0 ⇒ x = 1 … (iii) A(1,-3)


(1) (2)
Solving (i) and (iii), we get y= ± 3
C
Therefore, the circles (i) and (ii) intersect at A(1, 3) and B(1, − 3) . X’ O (2,0)
X

Area of enclosed region = Area OACBO = 2 Area OACO


B(1,-3)
= 2 [Area OAD + Area ACD]
Y’
1 2
= 2∫ 4 − (x − 2)2 dx + 2∫ 4 − x2 dx Figure 25.17
0 1

2 1
= 2∫ 4 − x2 dx + 2∫ 4 − (x − 2)2 dx
1 0

2 1
 x 4 − x2 4 x  (x − 2) 4 − (x − 2)2 4  x − 2   x 2 a2 x
2 2 2
=2 + sin−1  +2  + sin−1     a − x dx ⇒ a − x + sin−1 
 2 2 2  2 2  2 
   2 2 a 
 1  0

 3  π   3 π  8π
= 2 π − − 2  + 2 − − 2  + π = − 2 3 sq. units
   
 2  6   2 6  3

Illustration 15: Using integration, find the area of the region given below:
{(x, y): 0 ≤ y ≤ x2 + 1, 0 ≤ y ≤ x + 1, 0 ≤ x ≤ 2}  (JEE ADVANCED)
M a them a ti cs | 25.9

Sol: Same as above illustration, by solving given equation y = x2 + 1 and Y

1
x+
y = x + 1 we will be get their points of intersection and after that using
2
y=x +1

y=
integration method and taking these points as limit we can obtain required
F

area.
(1, 2)
C
The region is shaded as shown in the Fig. 25.18. B x=2
A
Given, y=x +1
2
… (i) (0, 1)

y = x + 1  … (ii) X
O D E
X

On solving (i) and (ii), we have x2 + 1 = x + 1 Figure 25.18


⇒ x = 0, 1 and y = 1, 2
∴ The shaded region can be divided into two parts OABCDO and CDEFC.
Limits for the area OABEO are x = 0 and x = 1.
Limits for the area EBDFE are x = 1 and x = 2.
Area of the shaded region = Area OABEO + Area EBDFE.
1 2
1 2 2  x3   x2  1  4 1  23
= ∫ (x + 1)dx + ∫ (x + 1)dx =  + x  +  + x  =  + 1  +  + 2 − − 1  = sq. units
 3  0  2 1  3  2 2  6
0 1

Illustration 16: Find the area of the following region: [(x, y): y2 ≤ 4x, 4x2+4y2 ≤ 9]  (JEE ADVANCED)

Sol: Similar to above problem, Here the required area is equal to Area AOBA + Area ACBA.
Given y2 = 4x … (i)
2
3
4x2 + 4y2 = 9 ⇒ x2 + y2 =    … (ii)
2
Y
1  1 
Curves (i) and (ii) intersect at A  , 2  and B  , − 2 
 2   2 
9
=
2

A
4y

2
1 y = 4x
Limits for the area OAB are x = 0, x =
+
4x 2

2
1 3 C
Limits for the area ACB are
= x = ,x . X’

( (
X
2 2 O x=1 3
,0
2 2
The required area = Area AOBA + Area ACBA
B
 1/2 3/2 9 
= 2  ∫ y1 dx + ∫ y 2 =
dx  2  ∫
1/2 3/2
4x dx + ∫ − x2 dx 
 0 1/2   0 1/2 4  Y’

1/2 3/2 Figure 25.19


2  x 9 9  x 
= 4  x3/2  + 2 . − x2 + sin−1  
3 0  2 4 8  3 / 2 1/2

8 1  1 9 1  4 1 9π 1 1 9 1
= . + 0 − +  sin−1 1 − sin−1   = − +  − sin−1  = + cos−1 .
3 2 2  2 4 3  3 2 2 42 3 3 2 4 3

Illustration 17: Draw a rough sketch and find the area of the region bounded by the two parabolas y2 = 8x and x2 = 8y,
by using method of integration. (JEE MAIN)
2 5 . 1 0 | Area Under the Cur ve and Linear Programming

Sol: As the given two equation is the equation of parabola


which intersect at O(0, 0) and A(8, 8), and the required area
2
Y

is equal to Area OBADO – Area OADO. A(8, 8)

Given parabolas are y2 = 8x


1
… (i) 2
x = 8y B
and, x2 = 8y … (ii)
The curves (i) and (ii) intersect at O(0, 0) and A(8, 8).
X’ X
∴ Required Area = Area OBADO – Area OADO O D

8
= ∫ (y1 − y 2 )dx
0

8
8 x2   x3/2 1 x3  64 2
y = 8x
= ∫  8x −  dx= 2 2 . −  = sq. units.
0 8  3 / 2 8 3  3 Y’
  0
Figure 25.20

Illustration 18: Find the area between the curves y = 2x, x + y = 1 and x-axis.  (JEE MAIN)
1 2
Sol: Here y = 2x and x + y = 1 is a two line intersect at p  ,  , therefore using integration method we can obtain
required area. 3 3

Given y = 2x … (i)


and, x + y = 1  … (ii)
Solving (i) and (ii), we get x + 2x = 1 ⇒ x = 1/3.
Line (i) intersects with the x – axis at the origin and the line (ii) intersects with the x – axis at x = 1.

P(x = 1/3)
x+
2x

y=
=

1
Y

O A
X
X=0 X=1/3 X=1

Figure 25.21

1
1/3 1 1/3  x2 
So required area = ∫0 =  x2  +  x − 
2x dx + ∫ (1 − x)dx
1/3  0  2 
 1/3

1 1  1 1  1
= + − −  = sq. units
9  2   3 18  3

Illustration 19: Using the method of integration, find the area of the region bounded by lines: 2x + y = 4, 3x – 2 y
= 6 and x – 3y + 5 = 0 (JEE ADVANCED)

Sol: Same as above problem.


Given equation of the lines are 2x + y = 4  … (i)
M a them a ti cs | 25.11

3x – 2y = 6 … (ii)
x – 3y + 5 = 0 … (iii)
Solving (i) and (ii), we get (2, 0)  Y
Solving (ii) and (iii), we get (4, 3)
0

2x
Solving (i) and (iii), we get (1, 2) + 5= (4,3)

+
- 3y

y=
X
4 x +5 4  3x − 6 

4
2 (1,2)
∴ Required Area = ∫1   dx − ∫1 (4 − 2x)dx − ∫2   dx

6
3   2 

y=
-2
4 4
1  x2  1  3x2 

3x
=  + 5x  − [4x − x2 ]12 −  − 6x  X
3  2 1 2  2  2 O (2,0)

1 1  1 Figure 25.22
= (8 + 20) −  + 5   − [(8 − 4) − (4 − 1)] − [(24 − 24) − (6 − 12)]
3 2  2

7
= = sq. units.
2

SKETCH OF STANDARD CURVES

2 2 2
Y x +y =a
x y
b + =1 (0 ,0)
a b

O a X

2
Y x = 4by
2
y = 4ax (b > 0)

2 (a > 0)
y = -4ax X
X

2
x = -4by

2 2
Y x +y Y
2 =1
2 2
2
a b x -y
2 2 =1
a b
X
O
X
O
2 5 . 1 2 | Area Under the Cur ve and Linear Programming

Y X Y
1 y = sinx 3
2
xy = c  2
 O x
-
O 2 -1 2
y = cosx

y=tanx Y
Y
y=cotx
y = logex


O  O 1
X
2
y=tanx y=cotx -

x
y=e
1

- O X

4. STANDARD AREAS
2
x = 4by
Y 2
y = 4ax

4.1 Area Bounded by Two Parabolas


X
Area between the parabolas y2 = 4ax and x2 = 4; a > 0, b > 0, is O k

16ab
| A |=
3
Figure 25.23

Illustration 20: Find the area bounded by y = x and x = y . (JEE MAIN)

Sol: By using above mentioned formula.


Y xx =
= yy
Area bounded is shaded in the figure
yy =
= xx
1 1
Here, a = and b =
4 4

\Using the above formula, Area = (16 ab)/ 3


X
O

Figure 25.24
16 × (1 / 4) × (1 / 4) 1
= =
3 3
M a them a ti cs | 25.13

4.2 Area Bounded By Parabola and a Line Y y = mx


2
y = 4mx
8a2
Area bounded by y = 4ax and y = mx; a > 0, m > 0 is A =
2

3m3
O
Area bounded by x2 = 4ay and =
y mx; a > m > 0 x=c
X

8a2
is =
y mx; a > m > 0 A =
3m3
Figure 25.25
Illustration 21: Find the area bounded by, x2 = y and y = |x|. (JEE MAIN)

8a2
Sol: Using above formula, i.e. A = Y
3m3

Area bounded is shaded in the Fig. 25.26.


Here, a = 1/4, m = 1 2
1
2×8×  x
 8a2  4 1 x=-c O x=c
\Using the above formula, Area =
= 2  =
 3m 
3
3 × (1)3 3
  Figure 25.26
Illustration 22: Find the area bounded by y = x and x = | y |. 
2
(JEE MAIN)
Y
Sol: Here, a = 1/4, m = 1, and required area is divided in to two equal parts at above and
below x – axis. x=c

 8a2 
Hence required area will be 2  . X
 3m3  O
 
 8a2  2 × 8 × (1 / 4 )
2 x=-c
1
\Using the above formula, Area = = 2  =
 3m3  3 × (1)3 3
 
Figure 25.27

4.3 Area Enclosed by Parabola and It’s Chord Y

Area between y = 4ax and its double ordinate at x = a is


2
D A
2
Area of AOB = (area ABCD)
3
O X

B
C
x=a

Figure 25.28

Illustration 23: Find the area bounded by y = 2x – x2, y + 3=0. (JEE MAIN)

Sol: Here first obtain area of rectangle ABCD and after that by using above
mentioned formula we will be get required area. D C
Solving y = 2x – x2, y + 3 = 0, we get x = –1 or 3 -1
X
3
Area (ABCD) = 4 × 4 = 16.
y = -3
A B
2 32
∴ Required area = × 16 =
3 3 Figure 25.29
2 5 . 1 4 | Area Under the Cur ve and Linear Programming

4.4 Area of an Ellipse (0, b)

x2 y2
For an ellipse of the form + 1 is
= A = πab (a, 0)
a2 b2

Figure 25.30
PLANCESS CONCEPTS

Try to remember some standard areas like for ellipse, parabola. These results are sometimes very helpful.

Vaibhav Gupta (JEE 2009 AIR 54)

5. SHIFTING OF ORIGIN
Area remains unchanged even if the coordinate axes are shifted or rotated or both. Hence shifting of origin /
rotation of axes in many cases proves to be very convenient in finding the area.
For example: If we have a circle whose centre is not origin, we can find its area easily by shifting circle’s centre.
Illustration 24: The line 3x + 2y = 13 divides the area enclosed by the curve 9x2 + 4y2 – 18x – 16y – 11 = 0 into two
parts. Find the ratio of the larger area to the smaller area.  (JEE ADVANCED)

Sol: Given 9x2+ 4y2 – 18x – 16y – 11 = 0 … (i)

and, 3x + 2y = 13 … (ii)

9(x2 – 2x) + 4 (y2 – 4y) = 11;

⇒ 9[(x – 1)2 – 1] + 4 [(y – 2)2 – 4] = 11

⇒ 9(x – 1)2 + 4(y – 2)2 = 36

(x − 1)2 (y – 2)2 X2 Y2
⇒ + 1⇒
= + 1
= (where X = x – 1 and Y = y – 2)
4 9 4 9 y
Hence 3x + 2y = 13 (0,3) 3x + 2y = 6
A1
⇒ 3(X + 1) + 2(Y + 2) = 13 Q
A2
⇒ 3X + 2Y = 6 P
x
O
X Y (2, 0)
⇒ + =1
2 3
∴ Area of triangle OPQ = 1/2 × 2 × 3 = 3
Figure 25.31
Also area of ellipse = π (semi major axes) (semi minor axis) = π.3.2 = 6π
6π 3π
A1 = − area of ∆OPQ = − 3
4 2

 6π  9π
A2 =3   + area of ∆OPQ = + 3
 4  2

A2 +3
2 3π + 2
Hence,
= =
A1 3π π−2
−3
2
M a them a ti cs | 25.15

6. DETERMINATION OF PARAMETERS
In this type of questions, you will be given area of the curve bounded between some axes or points, and some
parameter(s) will be unknown either in equation of curve or a point or an axis. You have to find the value of the
parameter by using the methods of evaluating area.

4
Illustration 25: Find the value of c for which the area of the figure bounded by the curves y = ; x = 1 and
9 x2
y = c is equal to .  (JEE MAIN)
4

Sol: By using method of evaluating area we can find out the value of c. Y
2
.c
1 1 c
 4  9  4 9 y=c
A= ∫ c − 2  dx = ;  cx +  =
x 4  x 2 4
2/ c 
c
9 9
(c + 4) − (2 c + 2 c ) =; c − 4 c + 4 =
4 4 X
O
9 3 3 x=1
⇒ ( c − 2)2 =⇒ ( c − 2) = or –
4 2 2 Figure 25.32
Hence c = (49/4) or (1/4)
Illustration 26: Consider the two curves:
C1 : y = 1 + cos x, and C2 : y = 1 + cos(x – α) for α ∈ (0, π/2) and x ∈ [0, p].
Find the value of α, for which the area of the figure bounded by the curves C1, C2 and x = 0 is same as that of the
area bounded by C2, y = 1 and x = π. For this value of α, find the ratio in which the line y = 1 divides the area of the
figure by the curves C1, C2 and x = π.  (JEE ADVANCED)

Sol: Solve C1 and C2 to obtain the value of x, after that by following given condition we will be obtain required
value of α.
Solving C1 and C2 , we get
α
1 + cos x = 1 + cos(x – α) ⇒ x = α – x ⇒ x =
2
According to the question, C1
A1 + A2 = A
α /2 π 2
∫ (cos x − cos(x − α ))dx = − ∫ (cos(x − α ))dx A1 C2
0 π

2 1
π y=1
α /2 +α
⇒ sinx − sin(x − α )=
 sin(x − α ) 2 A2
0 π
/2
O    
 α  α  π +
⇒ sin − sin  −   − [0 − sin( −α )] = sin   − sin( π − α ) 6 2 2
 2  2  2
Figure 25.33
α α π
⇒ 2sin − sin α =1 − sin α . Hence, 2sin = 1 ⇒ α=
2 2 3

7. AREA BOUNDED BY THE INVERSE FUNCTION


The area of the region bounded by the inverse of a given function can also be calculated using this method. The
graph of inverse of a function is symmetric about the line y = x. We use this property to calculate the area. Hence,
area of the function between x = a to x = b, is equal to the area of inverse function from f(a) to f(b).
2 5 . 1 6 | Area Under the Cur ve and Linear Programming

Illustration 27: Find the area bounded by the curve g(x), the x-axis and the lines at y = –1 and
x3 x2 13x
y = 4, where g(x) is the inverse of the function f(x) = + + +1 .  (JEE MAIN)
24 8 12

Sol: Here f(x) is a strictly increasing function therefore required area will be
2 0 Y
A=∫ (4 − f(x))dx + ∫ (f(x) + 1)dx

0 −2 6 f(x)

x3 x2 13x
Given f(x) = + + +1
24 8 12
f(1) = 6
⇒ f(0) = 1; f(2) = 4 and f(–2) = –1 (inflection
(0, 2)
point )
x2 x 13
Also, f ' (x) = + + , -1
8 4 12 O X
f(-1) = 2
i.e. f(x) is a strictly increasing function. -2

2 0 Figure 25.34
∴ A= ∫ (4 − f(x))dx + ∫ (f(x) + 1)dx
0 −2

2  0  3 
x3 x2 13x x x2 13x
A= ∫  24 8 12 
 4 − − − − 1  dx + ∫  24 8 + 12 + 1 + 1  dx
 +
0  −2  
 24 23 13.22     24 23 13.22   16
∴ A=  3.2 − − −  − (0) + (0) −  − + − 2.2  =
 24.4 8.3 12.2   24.4 8.3 12.2  3
     

Illustration 28: Let f(x) = x3 + 3x + 2 and g(x) is the inverse of it. Find the area bounded by g(x), the x-axis and the
ordinate at x = –2 and x = 6.  (JEE ADVANCED)

6
Sol: Let A = ∫ f −1 ( x ) dx Y
−2 4
Substitute x = f (u) or u = f −1 ( x )
f −1 ( 6 )
= ∫ u f −1 (u) du (0, 1)

f −1 ( 2 )
-2
O 2
X
-1
f −1 ( 6 )
= ∫ (
4 3u2 + 3 du ) Figure 25.35
f −1 ( 2 )

2 and f (1 ) = 6
We have, f ( −1 ) =
1 1

∫ u (3 u ) ∫ (3 u )
2 3
= + 3 du 2
= + 3u du
−1 0
1
3  9
=  u4 + 3u2  = Sq. units.
 2 0 2
M a them a ti cs | 25.17

8. VARIABLE AREA
If y = f(x) is a monotonic function in (a, b), then the area of the function y = f(x) bounded by the lines at x = a,
a+b
x = b, and the line y = f(c), [where c ∈ (a, b)] is minimum when c = .
2
c b Y
Proof: A = ∫ f(c) − f(x)dx + ∫ (f(x) − f(c))dx
y=f(x)
f(b)
a c
(0,f(c)) y=f(c )
c b f(a)
= f(c) (c − a) − ∫ (f(x))dx + ∫ (f(x))dx − f(c)(b − c)
a c
x
b c O x=a c x=b
= {(c – a) – (b – c)} f(c) + ∫ (f(x))dx − ∫ (f(x))dx
c a
Figure 25.36

b c
A = [2c − (a + b)] f(c) + ∫ (f(x))dx − ∫ (f(x))dx
c a
dA a+b a + b dA
For maxima and minima = 0 ⇒ f ‘ (c) = [2c – (a + b)] = 0 (as f ‘ (c) = 0) hence c = also c < , <0
dc 2 2 dc
a + b dA a+b
and c > , > 0 Hence A is minimum when c =
2 dc 2

9. AVERAGE VALUE OF A FUNCTION


In this section, we would study the average of a continuous function. This concept of average is frequently applied
in physics and chemistry.
b
1
b − a ∫a
Average of a function f(x) between x = a to x = b is given by y av = f(x)dx

PLANCESS CONCEPTS

(a) Average value can be positive, negative or zero .


b
1
b ∫0
(b) If the function is defined in (0, ∞), then y av = Lim f(x)dx provided the limit exists
b→∞
1
 1 b 2 2
(c) Root mean square value (RMS) is defined as ρ = ∫ f (x)dx 
 b − a a 

(d) If a function is periodic then we need to calculate average of function in particular time
period that is its overall mean.
Vaibhav Krishnan (JEE 2009 AIR 22)

Illustration 29: Find the average value of y2 w.r.t. x for the curve ay = b a2 − x2 between x = 0 & x = a. Also find
the average value of y w.r.t. x2 for 0 ≤ x ≤ a.  (JEE MAIN)
b
1
b − a ∫a
Sol: As average of a function f(x) between x = a to x = b is given by y av = f(x)dx
2 5 . 1 8 | Area Under the Cur ve and Linear Programming

a
b2 b2 2 2b2
Let f(x) = y =
2 (a2 − x2 ) Now f(x)
= |av x2 )dx
∫ (a − =
a2 a2 (a − 0) 0 3

a2 a2 a2
1 2 b 2 2 2 b 2 2ba3
Again yav w.r.t. x as f(x) |av
2
=
(a2 − 0)
∫ y d(x =
)
a2a
∫ a − x dx=
a3
∫ 2t dt= 3
0 0 0

10. DETERMINATION OF FUNCTION


Sometimes the area enclosed by a curve is given as a variable function and we have to find the function. The area
dAax
function Aax satisfies the differential equation = f(x) with initial condition Aaa = 0 i.e. derivative of the area
dx
function is the function itself. Thus we can easily find f(x) by differentiating area function.

PLANCESS CONCEPTS

Aax
If F(x) is integral of f(x) then, = ∫ f(x)dx
= [F(x) + c]

And since, Aaa= 0= F(a) + c ⇒ c = – F(a).

Aax F(x) − F(a) . Finally by taking x = b we get, =


∴ = Aba F(b) − F(a)
Note that this is true only if the function doesn’t have any zeroes between a and b.
If the function has zero at c then area = |F(b) – F(c)| + |F(c) – F(a)|
Vaibhav Gupta (JEE 2009 AIR 54)

Illustration 30: The area from 0 to x under a certain graph is given to be A = 1 + 3x − 1 , x ≥ 0 ;


(a) Find the average rate of change of A w.r.t. x and x increases from 1 to 8.
(b) Find the instantaneous rate of change of A w.r.t. x at x = 5.
(c) Find the ordinate (height) y of the graph as a function of x.
(d) Find the average value of the ordinate (height) y, w.r.t. x as x increases from 1 to 8.  (JEE ADVANCED)

Sol: Here by differentiating given area function we can obtain the main function.

A(8) − A(1) 3
(a) A(1) = 1, A(8) = 4 ; =
8 −1 7

dA 1. 3 3
(b)
= =
dx x =5 2 1 + 3x x =5 8

3
(c) y =
2 1 + 3x
8 8
1 3 1 3 3
(d) = ∫ dx = ∫ dx
(8 − 1) 1 2 1 + 3x 7 1 2 1 + 3x 7
M a them a ti cs | 25.19

Illustration 31: Let C1 & C2 be the graphs of the function y = x2 & y = 2x, 0 ≤ x ≤ 1 respectively. Let C3 be the graphs
of a function y = f(x), 0 ≤ x ≤ 1, f(0) = 0. For a point P on C1, let the lines through P, parallel to the axes, meet C2 &
C3 at Q & R respectively (see figure). If for every position of P(on C1), the area of the shaded regions OPQ & ORP
are equal, determine the function f(x).  (JEE ADVANCED)

Sol: Similar to the above mentioned method. C2:y=2x


h2 h
 y 2 (1/2,1) C1:y=x
2

∫ y −
2


dy =∫ (x − f(x))dx differentiate both sides w.r.t. h (0,1)
(1,1)
0 0 C1

 h2 
Q 2

 h −  2h = h2 – f(h)
P(h,h )
 2 

 h2 
f(h) = h2 –  h −  2h O
 2 
 (0,0) R

C3:y=f(x)
= h2 – h(2h – h2) = h2 – 2h2 + h3
Figure 25.37
f(h) = h – h 3 2

f(x) = x3 – x2 = x2(x – 1)

11. AREA ENCLOSED BY A CURVE EXPRESSED IN POLAR FORM


r = a (1 + cosθ) (Cardioid)
2π 2π
1 2 a2 θ
= 4
2 ∫0 ∫ 4 cos
A =r d θ dθ
2 0
2

θ
Substitute = t , dθ = 2dt
2
π
3πa2
A= a2 ∫ 4 cos4 t dt= 8 ×
0
16 Figure 25.38

Illustration 32: Find the area enclosed by the curves x = a sin3t and y = acos3t.  (JEE MAIN)

Sol:
2 2 2
a3 and dx = 3asin2 t.cos t.dt
x3 + y3 =
a π /2
A = 4 ∫ y dx ; A = 4a2 ∫ 3cos
3
t sin2 t cos t dt
0 0
a
π /2 2 2
1.3.1 π 12a π 3πa
=A 12a2 ∫ sin2 t cos=
4
t dt (12a2 ) . = . =
0
6.4.2 2 32 8

Figure 25.39
2 5 . 2 0 | Area Under the Cur ve and Linear Programming

Linear Programming

1. INTRODUCTION
Linear Programming was developed during World War II, when a system with which to maximize the efficiency of
resources was of utmost importance.

2. LINEAR PROGRAMMING
Linear programming may be defined as the problem of maximising or minimising a linear function subject to linear
constraints. The constraints may be equalities or inequalities. Here is an example.
Find numbers x1 and x2 that maximize the sum x1 + x2 subject to the constraints x1 ≥ 0, x2 ≥ 0, and
x1 + 2x2 ≤ 4
4x1 + 2x2 ≤ 12
−x1 + x2 ≤ 1
Here we have two unknowns and five inequalities (constraints). Notice that these constraints are all linear functions of
the variables. The first two constraints, x1 ≥ 0 and x2 ≥ 0, are special. These are called no negativity constraints and are
often found in linear programming problems. The other constraints are called the main constraints. The function to
be maximised (or minimized) is called the objective function. In the above example the objective function is x1 + x2 .

3. GRAPHICAL METHOD
As we have only two variables, we can solve this problem by plotting the constraints with x1 and x2 as axes. The
intersection region of these inequalities is called feasible region for the objective function. This is the region which
satisfies all the constraints. Now from this feasible region we have to select point(s) such that objective function is
maximized or minimized.

Theorem 1: Let R be the feasible region (convex polygon) for a linear programming problem and let Z=ax + by
be the objective function. When Z has an optimal value (maximum or minimum), where the variables x and y are
subject to constraints described by linear inequalities, this optimal value must occur at a corner point (vertex) of
the feasible region.

Theorem 2: Let R be the feasible region for a linear programming problem and let Z=ax + by be the objective
function. If R is bounded, then the objective function Z has both a maximum and a minimum value on R and each
of these occurs at corner point (vertex) of R.

Remark: If R is unbounded, then a maximum or a minimum value of the objective function may not exist. However,
if it exists it must occur at a corner point of R. (By Theorem 1).
So for the above example
6 4x1 + 2x2 = 12
Corner point (x1, x2) Z (= x1 + x2) value
-x1 + x2 = 1
0,1 1
3,0 3
2
8/3,2/3 10/3 1 -x1 + 2x2 = 4
2/3,5/3 7/3
3 4

Figure 25.40
M a them a ti cs | 25.21

Hence (8/3, 2/3) is the optimal solution.


Note that z has also minimum value in the feasible region at (0, 1).
This method of solving is generally called as corner point method. Note that a function can have more than one
optimal points.

4. MODELS
There are few important linear programming models which are more frequently used and some of them we
encounter in our daily lives.
(a) Manufacturing/Assignment problems: In these problems, we determine the number of units of different
products which should be produced and sold by a firm when each product requires a fixed manpower,
machine hours, labour hours per unit of product, warehouse space per unit of the output. In order to make
maximum profit.
Example: There are I persons available for J jobs. The value of person i working 1 day at job j is aij , for i =
1,……,I, and j = 1,…..,J. The problem is to choose an assignment of persons to jobs to maximize the total value.
An assignment is a choice of numbers, xij , for i = 1,……,I, and j=1,……,J, where xij represents the proportion of
person i’s time that is to be spent on job j. Thus,
J
∑ xij ≤ 1 For i = 1,……, I  ... (i)
j=1
I
∑ xij ≤ 1 For j=1,……, J  ... (ii)
i=1

And xij ≥ 0 for i = 1,……,I, and j=1,……,J  ... (iii)

Equation (i) reflects the fact that a person cannot spend more than 100% of his time working, (ii) means that
only one person is allowed on a job at a time, and (iii) says that no one can work a negative amount of time
I J
on any job, Subject to (i), (ii) and (iii) , we wish to maximize the total value of ∑∑ aij xij
=i 1 =j 1
(b) Diet problems: In these problems, we determine the amount of different kinds of nutrients which should
be included in a diet so as to minimise the cost of the desired diet such that it contains a certain minimum
amount of each nutrients.
Example: There are m different types of food, F1 ,........,Fm , that supply varying quantities of the n nutrients
, N1 ,........,Nn , that are essential to good health. Let c j be the minimum daily requirement of nutrient, N j
contained in one unit of food Fi . The problem is to supply the required nutrients at minimum cost.
Let y i be the number of units of food Fi to be purchased per day. The cost per day of such a diet is
b1 y1 + b2 y 2 + ......... + bm ym  … (i)
The amount of nutrient Nj contained in this diet is
a1 j y1 + a2 j y 2 + ......... + amj ym

For j = 1,…….., n. We do not consider such a diet unless all the minimum daily requirements are met, that is, unless

a1 j y1 + a2 j y 2 + ......... + amj ym ≥ c j For j = 1,…….., n  … (ii)



Of course, we cannot purchase a negative amount of food, so we automatically have the constraints

y1 ≥ 0, y 2 ≥ 0,................, ym ≥ 0  ... (iii)

Our problem is: minimize (i) subject to (ii) and (iii). This is exactly the standard minimum problem.
2 5 . 2 2 | Area Under the Cur ve and Linear Programming

(c) Transportation problems: In these problems, we determine a transportation schedule in order to find the
cheapest way of transporting a product from plants/factories situated at different locations to different
markets.
Example: There are I ports, or production plants, P1 ,..........PI , that supply a certain commodity, and there are
J markets, M1 ,..........MJ , to which this commodity must be shipped. Port Pi possesses an amount si of the
commodity (i=1,2,……I), and market M j must receive the amount rj of the commodity ( j = 1,……..J). Let bij be
the cost of transporting one unit of the commodity from port Pi to market M j . The problem is to meet the
market requirements at minimum transportation cost is
I J
∑∑ y ijbij  ... (i)
=i 1 =j 1
J
The amount sent from port Pi is ∑ yij ≤ yij and since the amount available at port Pi is si , we must have
j=1
J
∑ y ij ≤ si for i = 1,……,I  ... (ii)
j=1
I
The amount sent to market M j is ∑ yij , and since the amount required there is rj , we must have
i=1
I
∑ y ij ≤ rj for j = 1,……,I  ... (iii)
i=1
It is assumed that we cannot send a negative amount from PI to M j , we have
y ij ≥ 0 for I = 1,……..I and j =1,……..J.  ... (iv)
Our problem is minimize (i) subject to (ii), (iii) and (iv).

FORMULAE SHEET
b b
(a) Area bounded by a curve with x – axis: =
Area ∫=
y dx ∫ f(x)dx
a a
d d
(b) Area bounded by a curve with y – axis: Area = ∫ x dy = ∫ f(y)dy
c c
b t1
(c) Area of a curve in parametric form: Area = ∫ y dx = ∫ g(t)f '(t)dt
a t2
c b
(d) Positive and Negative Area: A = ∫ f(x)dx + ∫ f(x)dx ;
a c

(e) Area between two curves:


(i) Area enclosed between two curves intersecting at two different points.
b b
Area = ∫a (y1 − y 2 )dx = ∫a [f1 (x) − f2 (x)]dx
(ii) Area enclosed between two curves intersecting at one point and the x – axis.
α b
Area = ∫ f1 (x)dx + ∫ f2 (x)dx
a α

(iii) Area bounded by two intersecting curves and lines parallel to y – axis.
c b
Area = ∫ (f(x) − g(x))dx + ∫ (g(x) − f(x))dx
a c
M a them a ti cs | 25.23

(a) Standard Areas:


16ab
(i) Area bounded by two parabolas y2 = 4ax and x2 = 4by; a > 0, b > 0 : Area =
3
8a2
(ii) Area bounded by Parabola y2 = 4ax and Line y = mx : Area =
3m3
x2 y 2
(iii) Area of an Ellipse + 1 : Area = πab
=
a2 b2

Solved Examples

JEE Main/Boards is above the curve y = x2 y ≤ x ⇒ area is below the line


y=x
Example 1: Find area bounded by y = 4 – x2, x-axis and
the lines x = 0 and x = 2. Y

Sol: By using the formula of Area Y


Bounded by the x – axis, we can N(1, 1)
obtain
4

Required Area.
2 2
2
= ∫y =
dx ∫ (4 − x )dx
O
X
O M
0 0 2
2 1
 x3  8 16 1  x 2 x3  1
=  4x −  = 8 − = sq. units Area = ∫ (x − x )dx =2
 −  = sq. units
 3  3 3  2 3  6
 0 0  0

Example 2: Find the area bounded by the curve Example 4: Find the area of the region enclosed by
y2 = 2y – x and the y-axis. π
Y B y = sin x, y = cos x and x-axis, 0 ≤ x ≤ .
2
Sol: Here given equation is the 2 Y
equation of parabola with vertex
t Sol: Find point of intersection
(1, 1) and curve passes through is P. Therefore after obtaining
the origin. the co-ordinates of P and then P
integrating with appropriate
Curve is y2 – 2y = –x or (y – 1)2 = limits, we can obtain required
– (x – 1) Area. X
X O L /2
It is a parabola with O 2
At point of intersection P,
Vertex at (1, 1) and the curve passes through the origin. x=
π
as ordinates of y = sin
At B, x = 0 and y = 2 4
x and ; y = cos x are equal
Area
π 1 
2 2  y3 
2
4 Hence, P is  .  Required area
= ∫ x= 2
dy ∫ (2y − y ) dy =  y 2 −  = sq. units 4 2
 3  3
0 0  0 π/ 4 π /2
cos x dx = ( − cos x ) + ( sinx )
π/ 4 π /2
= ∫ sinx dx + ∫ 0 π/ 4
Example 3: Find the area of the region {(x, y): x ≤ y ≤ x}
2 0 π/ 4

 1   1 
Sol: Consider the function y = x2 and y = x Solving = − + 1  + 1 −  =2 − 2 sq. units
them, we get x = 0, y = 0 and x = 1, y = 1;x2 ≤ y ⇒ area  2   2
2 5 . 2 4 | Area Under the Cur ve and Linear Programming

Example 5: The area bounded by the continuous curve Example 8: Find the area bounded by the curves {(x, y) :
y = f(x), (lying above the x-axis), x-axis and the ordinates y ≥ x2, y ≤ |x|}
x = 1 and x = b is (b – 1) sin (3b + 4). Find f(x)
Sol: Here the region is symmetric about y-axis, the
Sol: Using Leibniz rule, we can solve given problem. required area is 2[area of shaded region in first
b quadrant].
∫ f(x)dx =
(b − 1)sin(3b + 4)
The curves intersect each other at x = 0 and x = ±1 as
1
Apply Leibniz Rule: differentiate both sides w.r.t. ‘‘b’’, shown in figure. The points of intersection are (–1, 1),
f(b) = sin (3b + 4) + 3(b – 1) cos (3b + 4) (0, 0) and (1, 1).
⇒ f(x) = sin (3x + 4) + 3(x – 1) cos (3x + 4) Since, the region is symmetric about y-axis, the required
area is 2[area of shaded region]
Example 6: Find the area bounded by the curve y = k sin 1 1
1 1  1
x and y = 0 from x = 0 to x = 2π. Hence, Area = 2∫ (x − x )dx = 2  x2 − x3  =
2
sq.
units. 0 2 3 0 3

Sol: Here the area of OAB is above the x-axis (y = 0) and


thus it is positive while the area BCD is below x-axis (y =
0) and in negative but equal in quantity. Example 9: Draw a rough sketch of the curve y = sin2x,
 π
Y x∈ 0,  . Find the area enclosed between the curve,
A  2 π
x-axis and the line x = .
2
B D π π π π
O  2
X Sol: Here by substituting x = 0, , , , we will
6 4 3 2
y = Sin x
get respective values of y. hence by plotting these
y = k sin x values we can draw the given curve.
π π
Some points on the sin2x graph are :
Area OAB = ∫ y dx= ∫ k sinx dx= k[ − cos x]0π
0 0
π π π π
= k[– cos p] – k [– cos 0] x 0
6 4 3 2
= k[– (–1)] – k [–(1)] = k + k = 2k y 0 0.25 0.5 0.75 1
∴ Total area = 4k sq. units. By plotting points and joining them, we trace the curve.
Area bounded by curve y = sin2x between x = 0 and
Example 7: Find the area bounded by the curve π
x= Y
x = a (θ – sin θ), y = a (1 – cos θ), 0 ≤ θ ≤ 2π, with x-axis. 2 A

Sol: Substitute the value of y and dx and integrate.


B
O X
θ= 2 π θ= 2 π /2
dx
∫ y dx
Area = = ∫ y dθ

=θ 0 =θ 0

2π Y’
= ∫ a(1 − cos θ)a(1 − cos θ)dθ π π π
2 2
0 12
= ∫ y=
dx ∫ sin2 x=
2 ∫0

dx (1 − cos2x)dx
= a2 ∫ (1 − 2cos θ + cos
2
θ)dθ 0 0
π
0
1 sin2x  2 1  π  
2π = x −  =  − 0  − (0 − 0)
 1  2 2  0 2  2  
= a2 ∫ 1 − 2cos θ + 2 (1 + cos2θ)
0 π
= sq.units

4
3 1 
= a2  θ − 2sin θ + sin2θ  = 3πa2 sq. units.
2 4 0
M a them a ti cs | 25.25

JEE Advanced/Boards Example 2: Let An be the area bounded by the curve


π
y = (tan x)n : n ∈ N and the lines x = 0, y = 0 and x = .
Example 1: A tangent is drawn to x2 + 2x – 4ky + 1 4
Prove that for n > 2, An + An–2 = and deduce
3 = 0 at a point whose abscissa is 3. The tangent is n−1
perpendicular to x + 3 = 2y. Find the area bounded by 1 1
< An < .
the curve, this tangent, x-axis and line x = –1 2n + 2 2n − 2

Sol: As we know multiplication of slopes of two Sol: We can write (tan x)n as tann−2 x (sec2 x − 1) .
perpendicular line is – 1, by using this, we can obtain π/ 4
the value of k and will get standard equation. After that tann−2 x (sec2 x − 1)dx we
using integration with respective limit, we will be get
Therefore by solving An= ∫
0
required area. can prove given equation.
Y
π/ 4
An = ∫ tann x dx : n > 2
0
A O B C
X π/ 4
3
= ∫ tann−2 x (sec2 x − 1)dx
-1
0
π /2
 tann−1 x 
 or An =   − An − 2
 n − 1  0
1
∴ An + An–2 =  … (i)
n−1
tann x ≤ tann–2 x
π
P (as 0 ≤ tan x ≤ 1 for 0 ≤ x ≤ )
4
dy x + 1 ⇒ An < An–2
x2 + 2x – 4ky + 3 = 0; = Tangent is perpendicular
dx 2k 1
to x + 3 = 2y ∴ An + An < An + An–2 = by (1)
n−1
x +11 1
∴ An <  … (ii)
∴   = −1 at x = 3 2(n − 1)
2k  2 
⇒ 1/k = –1 ⇒ k = –1 Similarly An + 2 < An

∴ Curve becomes (x + 1)2 = –4(y + 1/2) which is a ⇒ An + 2 + An < An + An


parabola with vertex at V(–1, –1/2). 1
or < 2An by (1)
Coordinates of P are (3, –9/2). (n + 2) − 1
Equation of tangent at P is y + 9/2 = –2(x – 3) 1
⇒ < An  … (iii)
2n + 2
B is (3/4, 0), C is (3, 0)
1 1
⇒ < An <
Required Area = Area (ACPV) – Area of triangle BPC. 2n + 2 2n − 2
3
x2 + 2x + 3 1
= ∫ dx − (BC)(CP) Example 3: A(a, 0) and B(0, b) are points on the ellipse
−4 2
−1 x2 y 2
3
+ =1 . Show that the area between the arc AB
1  x3  1 3 9 a2 b2
1
=  + x2 + 3x  −  3 −   
 3  and chord AB of the ellipse is ab (π – 2).
4  −1 2  4 2 4
Sol: Area between the chord and ellipse = Area
1  1  81 109 bounded by curve AB - Area of ∆ OAB.
=  27 + − (1 − 3)  − = sq. units.
4 3  16 48
b
Equation of line AB is : y =
− (x − a)
a
2 5 . 2 6 | Area Under the Cur ve and Linear Programming

b 2 1 
Equation of curve AB=
is y a − x2  , 0  on the x-axis.
a 2 
dy
Area of bounded region is It will have a turning points where =0
B dx
a
b  b  dy 1 3
∫  a a2 − x2 −  − (x − a)   dx ∴ = ( −3 − 4x) = 0 Þx = –
0  a  dx 2 4
d2 y 3
b a2 π a2  ( π − 2) Also = −4 . That is, it is a max. at x =
= 0 + −  = ab O A dx 2 4
a  4 2  4
Also it cuts y-axis where
Y
x = 0, then y = 1. Thus the
Alternate method:
shape of the curve is as B(0,1)
Area between the chord and ellipse = Area bounded by shown in the figure.
curve AB - Area of ∆ OAB
The required area is ABC. X
1 1
= πab − ab =
( π − 2) ab sq. units It is given by
A(-2,0) O

4 2 4 1/2 1/2
1
∫ y dx
= ∫ (2 − 3x − 2x2 )dx
Example 4: Find the area of the region bounded −2 −2
2
1 1/2
y = + 1, x =1 and tangent drawn at the point 1 3 2 2x3 
x = 2x − x − 
1 2  2 3 
P(2, 3/2) to the curve y = + 1. −2
x
1  1 31
2 3
Y 21
Sol: Here first obtain = 2   −   −    –
2 2 22 32 
equation of tangent and
N
M P(2.3/2)  
then use the formula for
area. 1 3 2 2 3 
2( −2) − ( −2) − ( −2 )
2 2 3
Equation of tangent at
y=1 
A
1 X
1  13  1  14  125
P(2, 3/2) to y = + 1 is O 1 2
=   −  −  = sq. units.
x 2  24  2  3  48
3 1
y− = − (x − 2) or x +
2 4
Example 6: Find the area of the region bounded by the
4y = 8.
curve x2 = 4y and x = 4y – 2.
Required area is area of region PMN
Sol: Solving given equation simultaneously, we will
2
 1  8−x get the point of intersection. Using these points as the
Area = ∫   + 1  −  dx
x 4  limits of integration, we calculate the required area.
1  
Y
2
 1 x2  5
=  lnx − x +  = ln2 − sq. units
 4 2  8
 1 (2, 1)
(-1, 1/4)
X
O
Example 5: Find the area of the region bounded by the 2
x x+2
1 The curve intersect each other, where = , or x2
x-axis and the curve y = (2 – 3x – 2x2). 4 4
2 – x – 2 = 0, or x = –1, 2
Sol: Here the curve will intersect the x-axis when y = 0, Hence, the points of intersection are (–1, 1/4) and (2, 1).
therefore by substituting y = 0 in the above equation we The region is plotted in figure. Since, the straight line
will get the points of intersection of curve and x – axis. x = 4y – 2 is always above the parabola x2 = 4y in the
1 interval [–1, 2], the required area is given by
⇒ 2 – 3x – 2x2 =0 or (2 + x) (1– 2x) = 0 or x = –2, x =
2 2
Thus, the curve passes through the points (–2, 0) and Area = ∫ [f(x) − g(x)]dx
−1
M a them a ti cs | 25.27

Sol: By solving these


2 2 Y
 x + 2 x2  1 1 2 1 3
Area = ∫  4 − 4  dx =  x + 2x − x 
4 2 3  −1 two equation
−1   P(a, a)
 simultaneously,
we can obtain their
B
1  8 1 1  9 intersection points and A
=  2 + 4 −  −  − 2 +   = sq. units.
4  3 2 3  8 then by subtracting
area of parabola from
O C X
area of circle we will
Example 7: Find by using integration, the area of the
be get the result.
ellipse ax2 + 2hxy + by2 = 1.
Y Solving the two equation, simultaneously we see that
Sol: The equation the two curves intersect at (0, 0), (a, a) and (a, –a). We
can be put in the form have to find the area of the region OAPBO, where P is
by2 + 2hxy + (ax2 – 1) = 0
L
y1 the point of intersection (a, a)
dx a
Cut an elementary strip. X
M O x y N Required area = ∫ [f(x) − g(x)]dx
Let the thickness of
2
0
strip = dx a a
= ∫ (2ax − x2 ) dx − ∫ ax dx
If y1, y2 be the values of y corresponding to any value 0 0
at x.
a a
Length of strip = y1 – y2 (2ax − x2 ) dx [a2 − (a − x)2 ] dx
Now, ∫ = ∫
0 0
2 2 2 2
= h x − b(ax2 − 1) = b − (ab − h2 )x2 To evaluate this integral, we substitute a – x = a sin θ
b b
and obtain
ab – h2 being positive here, since the conic is an ellipse. a 0

The extreme values of x, are given by ∫ x2 )dx


(2ax − = ∫ (acos θ)(−acos θ)dθ
0 π /2

b
y1 – y2 = 0, i.e., x = ± 2
π /2
2 21 π πa2
ab − h2 =∫ a cos =θ d θ a =
22 4
0
b/ab −h2 )
a
Hence, the area required = ∫ (y1 − y 2 )dx
Also =
a
2
a x3/2
2a2
− b/ab −h2 )
∫ (ax) dx =
3 3
0 0

b/(ab −h2 )
2  π 2
= ∫ b − (ab − h2 )x2 dx ∴ Required area = a2  −  sq. units
b  4 3
− b/(ab −h2 )

Example 9: Prove that the area of the region bounded


and putting (ab − h2 ) x = b sin θ , this becomes 5
by the curve a4y2 = x5(2a – x), is times to that of the
4
2
π /2
π circle whose radius is a.
∫ cos2 θ dθ = sq. units.
2
(ab − h ) −π /2 (ab − h2 )
Sol: The curve is a loop lying between the line x = 0 and
x = 2a and is symmetrical about the x-axis. Therefore
the required area Y
Example 8: Find the area of region lying above x-axis,
and included between the circle x2 + y2 = 2ax and the 2a

parabola y2 = ax. = 2 ∫ y dx
0
2a X
2 5/2
O x = 2a
=
a2
∫ x 2a − xdx
0
2 5 . 2 8 | Area Under the Cur ve and Linear Programming

To evaluate this integral, we put x = 2a sin2q. When, curves is not same.


1 4 5 
x = 0, θ = 0 and when x = 2a, θ = π ∴ Required area = 2  1 (x2 − 4)dx + (52 − x2 ) dx 
2 ∫
 2 4
∫ 
π /2 4
2 5/2 5
=
a2 0
∫ (2a) sin θ. 2a cos θ.4a sin θ cos θ dθ 4 5
2  x3  x 2 25 −1 x 
=  − 4x   + 2  25 − x + sin 
π /2 2 4  3 
  2 2 5 4
5.3.1.1 π 5a π 2
= 64a2 ∫ sin6 θ cos2 θ dθ = 64a2 . =
8.6.4.2 2 4
0 1  64  8 
=  − 16  −  − 8   +
5 2  3  3 
= x area of the circle whose radius is a.
4
 25 −1   25 −1 4  
2  0 + sin 1  −  6 + sin 
Example 10: Find the area bounded by the curves x2 +  2   2 5 
y2 = 25, 4y = | 4 – x2 | and y = 0.
1  32  4
=   + 25sin−1 1 – 12 − 25sin−1
Sol: Here x2 + y2 = 25 represent circle with centre at 2 3  5
origin and radius 5 unit. Therefore the required area
= 2 area ABC  16  π 4
=  − 12  + 25 − 25sin−1
 14 5   3  2 5
= 2  ∫ (4 − x2 )dx + ∫ 252 − x2 dx 
 2 4 4  20 π 4  −1 4 20 
= − + 25  −= sin−1   25cos −  sq. units.
Note: Here the portion is Y 3 2 5  5 3 
also bounded by two D
curves but we do not apply
B B(4,3)
A = ∫ [f(x) − g(x)]dx rule.
(-4,3) E
X
C’ A’ A C
Reason: Range of (-5,0) (-2,0) (2,0) (5,0)
integration of both the

JEE Main/Boards

Exercise 1 Q.3 Find the area bounded by the curve y = sinx, x-axis
and between x = 0, x = π.
Q.1 Write an expression for finding the area of the
shaded portion. Q.4 Write an expression for finding the area of the
shaded portion.
2
Y y=x +3 Y

3 x+y=5

X X
0 2 4 0

Q.2 Find the area bounded by the curve y = cos x, x-axis Q.5 Write an expression for finding the area bounded
and between x = 0, x = π. by the curve x2 = y and the line y = 2.
M a them a ti cs | 25.29

Q.6 Write an expression for finding the area of a circle Exercise 2


x2 + y2 = a2, above x-axis.
Single Correct Choice Type
Q.7 On sketching the graph of y = |x – 2| and evaluating
3 3 Q.1 The area of the figure bounded by the curve y = ex,
∫−1 | x − 2 | dx, what does ∫−1 | x − 2 | dx, represent on y = e–x and the straight line x = 1 is
the graph? 1 1
(A) e + (B) e −
e e
Q.8 Draw the rough sketch of the curve=y 3x + 4 and 1
find the area under the curve above x-axis and between (C) e + −2 (D) None of these
e
x = 0 and x = 4.

3 Q.2 The area bounded in the first quadrant by the


Q.9 Find the area under the curve y = above normal at (1, 2) on the curve y = 4x, x-axis & the curve
3
(1 − 2x)
x-axis and between x = –4 an x = –1. is given by
10 7 4 9
(A) (B) (C) (D)
Q.10 Find the area bounded by the curves y = 6x – x 2 3 3 3 2
and y = x2 – 2x.
Q.3 The area of the figure bounded by the curves
Q.11 Draw a rough graph of f(x)= x + 1 in the interval y = lnx and y = (lnx)2 is
[0, 4] and find the area of the region enclosed by the (A) e + 1 (B) e – 1 (C) 3 – e (D) 1
curve, x-axis and the lines x = 0 and x = 4.
Q.4 The area bounded by the curves y = x2 + 1 & the
Q.12 Find the area of the region bounded by the curve tangents to it drawn from the origin is:
xy – 3x – 2y – 10 = 0; x-axis and the lines x = 3, x = 4.
(A) 2/3 (B) 4/3 (C) 1/3 (D) 1

Q.13 Find the area bounded by the curve y = x sin x2, πx


Q.5 The area bounded by x2 + y2 – 2x= 0 & y = sin
π 2
x-axis and between x = 0 and x = . in the upper half of the circle is
2
π 4 π 2 8 π 2
(A) − (B) − (C) π − (D) −
Q.14 Using integration, find the area of the region 2 π 4 π π 2 π
bounded by the following curves, after making a rough
sketch: Q.6 Consider the region formed by the lines x = 0, y =
y = |x + 1| + 1, x = –2, x = 3, y = 0. 0, x = 2, y = 2. Area enclosed by the curves y = ex and
y = lnx, within this region is removed, then the area of
Q.15 Draw the rough sketch of y = sin 2x and determine the remaining region is
the area enclosed by the curve, the x-axis and the lines (A) 2(1 + 2  n2) (B) 2(2  n2 – 1)
x = π/4 and x = 3π/4.
(C) (2  n2 – 1) (D) 1 + 2  n2
Q.16 Find the area of the following region: {(x, y) :
x2 + y2 ≤ 2ax, y2 ≥ ax, x ≥ 0, y ≥ 0}. Q.7 The area bounded by the curves y = x(1 – lnx);
x = e–1 and positive x-axis between x = e–1 and x = e is
Q.17 Find the area bounded by the curve y2 = 4a2(x – 3)  e2 − 4e−2   e2 − 5e−2 
and the lines x = 3, y = 4a. (A)   (B)  
 5   4 
   
Q.18 Make a rough sketch of the region given below
 4e2 − e−2   5e2 − e−2 
and find its area using integration. {(x,y): 0 ≤ y ≤ x2 + 3 ; (C)   (D)  
 5   4 
0 ≤ y ≤ 2x + 3, 0 ≤ x ≤ 3}.    

Q.19 Determine the area enclosed between the curve


y = 4x – x2 and the x-axis.
2 5 . 3 0 | Area Under the Cur ve and Linear Programming

Q.8 The positive values of the parameter ‘a’ for which Q.15 The area of the region for which 0 < y < 3 – 2x – x2
the area of the figure bounded by the curve y = cos ax, & x > 0 is
π xπ 3 3
y ==0, x = ,x is greater than 3 are (A) ∫ (3 − 2x − x2 )dx (B) ∫ (3 − 2x − x2 )dx
6a 2a
1 0
(A) f (B) (0, 1/3)
1
(C) (3, ∞) (D) None of these (C) ∫ (3 − 2x − x2 )dx (D) None of these
0

Q.9 The value of ‘a’ (a > 0) for which the area bounded Q.16 The graphs of f(x) = x2 and g(x) = cx3
x 1 1 1 
by the curves y =+ , y = 0, x = a and x = 2a has (c > 0) intersect at the points (0, 0) &  ,  . If the
6 x2
 c c2 
the least value, is
region which lies between these graphs & over the
(A) 2 (B) 2 (C) 21/3 (D) 1 interval [0, 1/c] has the area equal to 2/3 then the value
of c is
Q.10 The ratio in which the area enclosed by the curve
(A) 1 (B) 1/3 (C) 1/2 (D) 2
 π
y = cos x  0 ≤ 0 ≤  in the first quadrant is divided by
 2
Q.17 The curvilinear trapezoid is bounded by the
the curve y = sin x, is
curve y = x2 + 1 and the straight lines x = 1 and x =
2. The co-ordinates of the point (on the given curve)
(A) ( 2 − 1) : 1 (B) ( 2 + 1) : 1
with abscissa xÎ[1, 2] where tangent drawn cut off from
(C) 2 :1 (D) 2 +1 : 2 the curvilinear trapezoid are ordinary trapezium of the
greatest area, is
Q.11 The area bounded by the curve y = f(x), the (A) (1, 2) (B) (2, 5)
x
co-ordinate axes & the line x = x1 is given by x1. e 1 .
Therefore f(x) equals  3 13 
(C)  ,  (D) None of these
(A) ex (B) x ex (C) xex – ex (D) xex + ex 2 4 

Q.18 In the given figure, if A1 is the area of the ∆AOB


Q.12 The area bounded by the curves y =− −x and and A2 is the area of the parabolic region AOB then the
x =− − y where x, y ≤ 0 A
ratio 1 as a → 0 is
(A) Cannot be determined A2
Y
(B) Is 1/3
2
A y=a
B
(C) Is 2/3
2
(D) Is same as that of the figure bounded by the curves y=x

y= −x ; x ≤ 0 and x =− y ; y ≤ 0
0
Q.13 The area from 1 to x under a certain graph is given
by A = (1 + 3x)1/2 – 1, x ≥ 0.. The average value of y w.r.t. (A) 1 (B) 8/9 (C) 3/4 (D) 2/3
x as x increases from 1 to 8 is
(A) 3/7 (B) 1/2 (C) 3/8 (D) 4/7
Previous Years’ questions
Q.14 The slope of the tangent to a curve Q.1 The area of the quadrilateral formed by the
y = f(x) at (x, f(x)) is 2x + 1. If the curve passes through tangents at the end points of latus rectum to the ellipse
the point (1, 2) then the area of the region by the curve, x2 y 2
the x-axis and the line x = 1 is + = 1, is  (2003)
9 5
(A) 5/6 (B) 6/5 (C) 1/6 (D) 1
(A) 27/4 sq. unit (B) 9 sq. unit
(C) 27/2 sq. unit (D) 27 sq. unit
M a them a ti cs | 25.31

Q.2 The area bounded by the curves= y x ,2y =


+3 x 5 15 9 7
(A) (B) (C) (D)
and x-axis in the 1st quadrant is  (2003) 64 64 32 32
(A) 9 sq. unit (B) 27/4 sq. unit
Q.10 The area (in square units) bounded by the curves
(C) 36 sq. unit (D) 18 sq. unit
=y + 3 0, x-axis, and lying in the first
x ,2y − x =
Q.3 The area enclosed between the curves y = ax2 and quadrant is  (2013)
x = ay2 (a > 0) is 1 sq unit. Then, the value of a is (2004) 27
(A) 36 (B) 18 (C) (D) 9
1 1 1 4
(A) (B) (C) 1 (D)
3 2 3 Q.11 The area of the region described by A

Q.4 The area of the equilateral triangle, in which three {


= (x, y) : x2 + y 2 ≤ 1 and y 2 ≤ 1 − x is } (2014)
coins of radius 1 cm are placed, as shown in the figure, π 4 π 4
(A) + (B) −
is  (2005) 2 3 2 3

sq.cm
(A) (6 + 4 3)sq cm π 2 π 2
(C) − (D) +
2 3 2 3
sq.cm
(B) (4 3 − 6)sq cm
x −1 y +1 z −1
sq.cm
(C) (7 + 4 3)sq cm Q.12 If the lines = = and
2 3 4
x −3 y −k z
= = intersect, then k is equal to  (2012)
sq.cm
(D) 4 3 sq cm 1 2 1

2 9
Q.5 The area enclosed within the curve |x| + |y| = 1 (A) -1 (B) (C) (D) 0
is ……..  (1981) 9 2

y
Q.13 The area bounded between the parabolas x2 =
Q.6 The area of the triangle formed by the positive 4
x-axis and the normal and the tangent to the circle x2 + and x2 = 9y and the straight line y = 2 is  (2012)
y2 = 4 at (1, 3) is …….. (1989) 10 2 20 2
(A) 20 2 (B) (C) (D) 10 2
3 3
Q.7 The area bounded by the curves y = (x – 1)2,

y = (x + 1)2 and y =
1
is  (2005) Q.14 The area bounded by the curves y = cos x and
4 3π
y = sinx between the ordinates
= x 0=
and x
1 2 2
(A) sq. unit (B) sq. unit
3 3 (2010)
1 1 (A) 4 2 + 2 (B) 4 2 − 1
(C) sq. unit (D) sq. unit
4 5
(C) 4 2 + 1 (D) 4 2 − 2
Q.8 The area (in sq. units) of the region

{( x, y ) : y 2 2 2
}
≥ 2x and x + y ≤ 4x, x ≥ 0, y ≥ 0 is: (2016) Q.15 The area of the region enclosed by the curves
1
8 4 2 =y x,= x e,= y and the positive x-axis is (2011)
(A) π − (B) π − x
3 3
3
π 2 2 4 (A) 1 sq. unit (B) sq. units
(C) − (D) π − 2
2 3 3
5 1
(C) sq. units (D) sq. units
Q.9 The area (in sq. units) of the region described by 2 2

{( x, y ) : y 2
}
≤ 2x and y ≥ 4x − 1 is :  (2015)
2 5 . 3 2 | Area Under the Cur ve and Linear Programming

JEE Advanced/Boards

Exercise 1 curve y = f(x) and compute the area enclosed by the


curve and the pair of tangents.
Q.1 In the adjacent figure, graphs of two functions
y = f(x) and y = sinx are given. y = sinx intersects, Q.4 Show that the area bounded by the curve
y = f(x) at A(a, f(a)); B(π, 0) and C(2π, 0). Ai (i = 1, 2, 3) logx − c
is the area bounded by the curve y = f(x) and y = sin y= , the x-axis and the vertical line through
x
x between x = 0 and x = a; i = 1, between x = a and
the maxima point of the curve is independent of the
x = π; i = 2, between x = π and x = 2π; i = 3.
constant c.
Y
y = f (x) Q.5 Consider the curve y = xn where n > 1 in the 1st
A2 quadrant. If the area bounded by the curve, the x-axis
A (2,0)
A1 and the tangent line to the graph of y = xn at the point
y=sinx B
 C (1, 1) is maximum then find the value of n.
O x=a x
y=sinx
Q.6 For what value of ‘a’ is the area of the figure
1 1
A3 bounded by the lines= y = ,y , x = 2 &
x 2x − 1
4
x = a equal to ln ?
5
If Ai = 1 – sin a + (a – 1) cos a, determine the function
f(x). Hence determine ‘a’ and A1. Also calculate A2 and A3. 1
Q.7 For the curve f(x) let two points on it are
1 + x2
Q.2 The figure shows two regions in the first quadrant.  1  1 
A(α,f(α)), B  − ,f '  −   (α > 0). Find the minimum
2 2  α  α 
Y y=sin x P(t, sin t ) area bounded by the line segments OA, OB and f(x),
where ‘O’ is the origin.
A(t)
X’ X
O Q.8 Find the area bounded by the curve y = sin−1 x and
π
Y’ the lines x = 0, y = .
2
2
Y P(t, sin t )
Q.9 If f(x) is monotonic in (a, b) then prove that the area
bounded by the ordinates at x = a ; x = b ; y = f(x) and
A(t) a+b
X’ X y = f(c), c ∈ (a, b) is minimum when c = . Hence
O 2
x3
Y’ if the area bounded by the graph of f(x) = − x2 + a,
3
A(t) is the area under the curve y = sin x2 from 0 to t the straight lines x = 0, x = 2 and the x-axis is minimum
and B(t) is the area of the triangle with vertices O, P and then find the value of ‘a’.
A(t)
M(t, 0). Find Lim .
t → 0 B(t) Q.10 Let ‘c’ be the constant number such that c > 1.
If the least area of the figure given by the line passing
Q.3 A polynomial function f(x) satisfies the condition
through the point (1, c) with gradient ‘m’ and the
f(x + 1) = f(x) + 2x+ 1. Find f(x) if f(0) = 1. Find also the
parabola y = x2 is 36 sq. units find the value of (c2 + m2).
equations of the pair of tangents from the origin on the
M a them a ti cs | 25.33

Q.11 Let C1 & C2 be two Q.17 Find the values of m(m > 0) for which the area
curves passing through the Y
C2
C bounded by the line y = mx + 2 and x = 2y – y2 is,
origin as shown in the figure. C1 (i) 9/2 square units & (ii) minimum. Also find the
A curve C is said to ‘‘bisect minimum area.
the area’’ the region between A
C1 & C2, if for each point P of B Q.18 Find the area bounded by the curve y = x e–x ;
C, the two shaded regions A xy = 0 and x = c where c is the x-coordinate of the
& B shown in the figure have X curve’s inflection point.
equal area. Determine the
O

upper curve C2, given that the bisecting curve C has the Q.19 Find the area of the region
equation y = x2 & that the lower curve C1 has the
equation y = x2/2. {( x, y ) : y 2
≤ 4 x, 4x2 + 4y 2 ≤ 9 }
Q.12 Consider one side AB of a square ABCD, (read in Q.20 For what value of ‘a’ is the area bounded by the
order) on the line y = 2x – 17 and the other two vertices curve y = a2x2 + ax + 1 and the straight line y = 0, x = 0
C, D on the parabola y = x2. & x = 1 the least?

(i) Find the minimum intercept of the line CD on y-axis. 1


Q.21 Consider two curves C1: y = and C2: y = ln x
(ii) Find the maximum possible area of the square ABCD. x
on the xy plane. Let D1 denotes the region surrounded
(iii) Find the area enclosed by the line CD with minimum by C1, C2 and the line x = 1 and D2 denotes the region
y-intercept and the parabola y = x2. Consider the two surrounded by C1, C2 and the line x = a. If D1 = D2. Find
curves C1 : y = 1 + cos x & C2 : y = 1 + cos (x – α) for the value of ‘a’.
α ∈ (0, π/2) ; x ∈ [0, p]. Find the value of α, for which
the area of the figure bounded by the curves C1, C2 &
x = 0 is same as that of the figure bounded by C2, y = 1 & x Exercise 2
= π. For this value of α, find the ratio in which the line y = 1
divides the area of the figure by the curves C1, C2 & x = π. Single Correct Choice Type

Q.1 The area bounded by the curve y = x2 – 1 & the


Q.13 Draw the rough sketch of y 2= x + 1 and
straight line x + y = 3 is
y 2 =−x + 1 and determine area enclosed by the two
curves. 9 7 17 17 17
(A) (B) 4 (C) (D)
2 2 6
Q.14 Let f : (0, ∞) → R be a continuous and strictly
x Q.2 The area bounded by the curve y = e–x & the lines
3 2
increasing function such that f (x) = ∫ t f (t)dt, ∀ x > 0. y = e–4 & x = 1 is given by
0
Find the area enclosed by y = f(x), the x-axis and the e3 − 4 e3 + 4
(A) (B)
ordinate at x = 3. e4 e4

Q.15 For what values of a Î[0, 1] does the area of the e3 + 1


(C) (D) None of these
figure bounded by the graph of the function y = f(x) e4
and the straight lines x = 0, x = 1 & y = f(a) is at a
minimum & for what values it is at a maximum if f(x) = y
Q.3 Area common to the curve = 9 − x 2 & x2 + y2
= 6x is
1 − x2 . Find also the maximum & the minimum area.

 π π+ 3 π− 3
(A) (B)
Q.16 Let f(x) = sin x ∀ x ∈ 0,  4 4
 2
π    3 3
f(x) + f(π – x) = 2 ∀ x ∈  , π  and f(x) = f(2π – x) ∀ x 3
2  (C) 3  π +  (D) 3  π + 
 4  4 
∈ (π, 2π).  

If the area enclosed by y = f(x) and x-axis is aπ + b, then


find the value of (a2 + b2).
2 5 . 3 4 | Area Under the Cur ve and Linear Programming

3 Q.11 The ratio in which the curve y = x2 divides the


Q.4 The area bounded by y = 2 – |2 – x| & y = is
|x|  πx 
region bounded by the curve ; y = sin   & the x-axis
4 + 3n3 4 − 3n3  2 
(A) (B) as x varies from 0 to 1, is
2 2
3 1 (A) 2 : p (B) 1 : 3
(C) + n3 (D) + n3
2 2 (C) 3 : p (D) (6 – π) : p

Q.5 The area bounded by the curves y = sin x & y = cos Q.12 Area of the region enclosed between the curves
x between x = 0 & x = 2 π is
x = y2 – 1 and
= x | y | 1 − y 2 is

(A) ∫ (sinx − cos x)dx (B) 2 2 sq. unit (A) 1 (B) 4/3 (C) 2/3 (D) 2
0

(C) 0 (D) 4 2 sq. unit Q.13 Let y = g(x) be the inverse of a bijective mapping f :
R → R, f(x) = 3x3 + 2x. The area bounded by the graph
of g(x). The x-axis and the ordinate at x = 5 is
Q.6 If f(x) = –1 + |x – 2|, 0 ≤ x ≤ 4 ; g(x) = 2 – |x|, –1 ≤ x
≤ 3. (A) 5/4 (B) 7/4 (C) 9/4 (D) 13/4

Then the area bounded by y = gof(x); x = 1, x = 4 and


x-axis is
Previous Years’ Questions
(A) 7/2 sq. units (B) 9/4 sq. units
(C) 9/2 sq. units (D) None of these Q.1 The area of the region between the curves
1 + sinx 1 − sinx
y= and y = and bounded by the
Q.7 Area enclosed between the curve y = sec x, y = –1 cos x cos x
cosec–1 x and the line x = 1 is π
lines x = 0 and x = is  (2008)
4
(A) ln (3 + 2 2) (B) ln (3 + 2 2) − 1
2 −1 t
(C) ln (3 + 2 2) − π / 2 (D) None of these
(A) ∫0 dt
(1 + t ) 1 − t2
2

Q.8 The area of the closed figure bounded by y = x, 2 −1 4t


(B) ∫ dt
0
y = –x the tangent to the curve=
y 2
x − 5 at the point (1 + t ) 1 − t2
2

(3, 2) is
2 +1 4t
15
(C) ∫0 dt
(A) 5 (B) (1 + t ) 1 − t2
2
2
35 2 +1 t
(C) 10 (D) (D) ∫ dt
2 0
(1 + t ) 1 − t2
2

Q.9 The line y = mx bisects the area enclosed by the


3 Q.2 Let the straight line x = b divide the area enclosed
curve y = 1 + 4x – x2 & the lines x = 0, x = & y = 0, by y = (1 – x)2, y = 0 and x = 0 into two parts R1(0 ≤ x
then m is equal to 2
1
13 6 3 ≤ b) and R2(b ≤ x ≤ 1) such R1 – R2 = . Then, b equals
(A) (B) (C) (D) 4  4 (2011)
6 13 2
3 1 1 1
(A) (B) (C) (D)
Q.10 The area common to y ≥ x & x > − y and the 4 2 3 4
curve x2 + y2 = 2 is
π 1 3π 1 π
(A) + (B) (C) (D)
4 3 2 3 2
M a them a ti cs | 25.35

Q.3 Let f : [–1, 2] → [0, ∞) be a continuous function such  4a2 4a 1 f( −1) 3a + 3a 
2
2    
that f(x) = f(1 – x) for all xÎ[–1, 2], Let R1 = ∫−1 x f(x)dx  
Q.12 If  4b2 4b 1  f(1)  = 3b 2
+ 3b ,
and R2 be the area of the region bounded by y = f(x), x  2
4c

4c 1 f(2)  3c2 + 3c 
     
= –1, x = 2 and the x-axis. Then, (2011)
f(x) is a quadratic function and its maximum value
(A) R1 = 2R2 (B) R1 = 3R2
occurs at a point V. A is a point of intersection of y = f(x)
(C) 2R1 = R2 (D) 3R1 = R2 with x-axis and point B is such that chord AB subtends
a right angled at V. Find the area enclosed by f(x) and
chord AB.  (2005)
Q.4 Find the area bounded by the curve x2 = 4y and the
straight line x = 4y – 2. (1981)
Q.13 A curve passes through (2, 0) and the slope of
Q.5 Find the area bounded by the x-axis, part of the (x + 1)2 + y − 3
tangent at point P(x, y) equals . Find the
 8  (x + 1)
curve = y  1 +  and the ordinates at x = 2 and
 x2  equation of the curve and area enclosed by the curve
x = 4. If the ordinate at x = a divides the area into two and the x-axis in the fourth quadrant.  (2004)
equal parts, find a.  (1983)
Q.14 Area of the region
Q.6 Find the area of the region bounded by the x-axis
π π
{( x, y ) ∈ R 2
:y≥ x + 3 , 5y ≤ x + 9 ≤ 15 } is equal to
and the curves defined by y = tan x, − ≤ x ≤ and  (2016)
3 3
π π 1 4 3 5
y = cot x, ≤ x ≤ (1984) (A) (B) (C) (D)
6 2 6 3 2 3

Q.7 Sketch the region bounded by the curves Q.15 Let S be the area of the region enclosed by
2 2
y
= 5−x and y = |x – 1| and find its area. (1985) y e− x =
= x 0 , and x = 1 . Then 
, y 0,= (2012)

1 1
Q.8 Find the area bounded by the curves x2 + y2 = 4, (A) S ≥ (B) S ≥ −
e e
x2 = – 2y and x = y.  (1986)
1 1  1 1  1 
(C) S ≤ 1 +  (D) S ≥ + 1 − 
4 e 2 e 2
Q.9 Find the area of the region bounded by the curve C :
π
y = tan x, tangent drawn to C at x = and the x-axis.
 4 (1988)

Q.10 Find all maxima and minima of the function y = x


(x – 1)2, 0 ≤ x ≤ 2.
Also, determine the area bounded by the curve y=x(x – 1)2,
the y-axis and the line x = 2. (1989)

Q.11 Compute the area of the region bounded by the


logx
curves y = ex log x and y = where log e = 1
ex
 (1990)
2 5 . 3 6 | Area Under the Cur ve and Linear Programming

PlancEssential Questions
JEE Main/Boards JEE Advanced/Boards

Exercise 1 Exercise 1
Q.4 Q.10 Q.17 Q.1 Q.5 Q.12
Q.14 Q.16 Q.20
Exercise 2 Q.21
Q.2 Q.6 Q.10
Q.12 Q.15 Q.17
Exercise 2
Q.18 Q.2 Q.3 Q.7
Q.11 Q.13
Previous Years’ Questions
Previous Years’ Questions
Q.2 Q.4 Q.7
Q.1 Q.3 Q.7
Q.10 Q.11 `Q.13

Answer Key

JEE Main/Boards

Exercise 1

74 3
Q.1 sq.units Q.2 2 sq. units Q.3 2 sq. units Q.4 ∫1 (5 − y)dy
3

2 a 112
Q.5 2∫0 y dy Q.6 ∫−2 a2 − x2 dx Q.7 5
 sq. units Q.8 sq. units
9

2 64 28
Q.9 sq. units Q.10 sq. units Q.11 sq. units
27 3 3

1
Q.12 (3 + 16 log 2) sq. units Q.13 sq. units Q.14 13.5 sq. units Q.15 1 sq. units
2

a2 16a 50 32
Q.16 (3π – 8) sq. units Q.17 sq. units Q.18 sq. units Q.19 sq. units
12 3 3 3
M a them a ti cs | 25.37

Exercise 2
Single Correct Choice Type

Q.1 C Q.2 A Q.3 C Q.4 A Q.5 A Q.6 B

Q.7 B Q.8 B Q.9 D Q.10 C Q.11 D Q.12 B

Q.13 A Q.14 A Q.15 C Q.16 C Q.17 C Q.18 C

Previous Years’ Questions


Q.1 D Q.2 A Q.3 A Q.4 A Q.5 2 sq. units

Q.6 2 3 sq. units Q.7 A Q.8 A Q.9 C Q.10 D Q.11 A

Q.12 C Q.13 C Q.14 D Q.15 B

JEE Advanced/Boards

Exercise 1

Q.1 f(x) = x sin x, a = 1 ; A1 = 1 – sina ; A2 = π – 1 – sina ; A3 = (3π – 2) sq. units Q.2 2/3

2 1
Q.3 f(x) = x2 + 1;y = ± 2x ; A = sq. units Q.4 Q.5 2 + 1 Q.6 a = 8
3 2
( π − 1) 2
Q.7 Q.8 2 Q.9 a = Q.10 104 Q.11 (16/9)x2
2 3
32 3 8
Q.12 (i) 3 ; (ii) 1280 sq. units ; (iii) sq. units Q.13 3 Q.14 Q.15= sq. units
3 2 3
7
3m + 2m2 +  2 9π 9 1
Q.16 4 Q.17 A 6 Q.18 1 – 3e–2 Q.19=  + − sin−1  sq. units
m3  6 8 4 3 
Q.20 a = –3/4 Q.21 e

Exercise 2
Single Correct Choice Type

Q.1 D Q.2 A Q.3 D Q.4 B Q.5 D Q.6 C Q.7 C


Q.8 A Q.9 A Q.10 A Q.11 D Q.12 D Q.13 B

Previous Years’ Questions


Q.1 B Q.2 B Q.3 C Q.4 9/8 sq. units Q.5 2 2

1 5π 1 1  1
Q.6 loge 3 sq. units Q.7 − sq. units Q.8 − π sq. units Q.9  log 2 −  sq. units
2 4 2 3  4
e2 − 5
Q.10 10/3 sq. units Q.11 sq. units Q.12 125/3 sq. units Q.13 4/3 sq. units
4e
Q.14 3/2 sq. units Q.15 A, B, D
2 5 . 3 8 | Area Under the Cur ve and Linear Programming

Solutions

JEE Main/Boards 3
Sol 7: ∫ x − 2 dx is the area under curve |x – 2|
Exercise 1 −1

where x ∈ [–1, 3]
4 4
x3 2 3
Sol 1: Area = ∫ x2 + 3 = + 3x 2 3
x2 x2
2
3 A = ∫ 2 − x + ∫ x − 2 = 2x − + − 2x
2
−1 2
2 2
−1 2
64 8 56 74 1 9
= + 12 − − 6 ⇒ Area = + 6 = sq. units ⇒ A = 2 + 2 + + – 6 – 2 + 4 = 5 sq. units
3 3 3 3 2 2
π /2 y
π /2
Sol 2: 2 ∫ cos xdx = 2 sinx 
0
= 2[1 – 0] = 2 sq. units
0

π
π
Sol 3: ∫ sinxdx =  − cos x  = − cos π + cos0 = 1 + 1 -1 2 3
x
0
0

= 2 sq. units

y = 3x+4
Sol 4: x + y = 5 ⇒ x = –y + 5
3
2 3  y2  9 1 Sol 8: y = 3x + 4 
⇒ ∫ x = ∫ ( − y + 5 ) dy =  − + 5y  = − 15 − + 5
4 1  2 1 2 2
4
= 6 sq. units
3
Expression = ∫ (5 − y ) dy
1
4
2 ( 3x + 4 )
3/2 
4 4
  0
Sol 5: y = x2 ; A = ∫=
y dx ∫ 3x + 4 dx = 
Y=2
0 0 9x3

Y=0 2  3/2 2
= 16 − 43/2  =  43 − 23 
9   9 
2 2 56 × 2 112
x= y A = 2∫ ydy = 64 − 8=
 = sq. units
9 9 9
0

3
Sol 6: Sol 9: y = above x axis & x ∈ [–4, –1]
(1 − 2x )
3

−1
−1  
3dx 3 1
∫ =  × 
-a a
−4 (1 − 2x )
3 
 (1 − 2x )
+2
( )( ) 
−2 −2
−4

−1
 
3 1 31 1 3× 8 2
=  = 
 − = = sq. units
4  1 − 2x  4 3 4 81 27
( ) ×
2 2 2
a 9 
a2 − x2 dx   −4
Area = ∫
−a
M a them a ti cs | 25.39

Sol 10: dt
xdx =
2
π /2 π /2
dt  cos t  −0 + 1 1
∫ sint = −  = = sq. units
3 4
0
2  2 0 2 2

Sol 14:

4

∫ ( 6x − x )
2
Area = − x2 + 2x dx
0

4
 4 2x3  2 64
= 8x − 2x 2
=  4x2 −  = 64 – × 64 = sq. units
 0  3  3 3
 0
f(x) = 1+ x
3 3 −1
Sol 11: f(x) = 1 + x 
∫ x + 1 + 1= ∫ x + 1 + 1 + ∫ −1 – x + 1
1 −2 −1 −2
4 4 4
 2x3/2  3 −1
∫ y dx= ∫ x + 1=  + x a x2  x2 
 3 = + 2x
0 − 
0 0  0
2  2  −2
−1
2 16 28
= × 23 + 4 = + 4 = sq. units 9 1 1 7 27
3 3 3 = + 6 − + 2 − + 2 = + 10 = sq. units
2 2 2 2 2

Sol 12: xy – 3x – 2y – 10 = 0
Sol 15:
3x + 10
y=
x−2
y



2
x
3 4
π /2
2 π /2
Area = 2 ∫ sin2x dx
=
2
 − cos2x 
π/ 4
π/ 4

= |+1 – 2| = 1 sq. units


4 4
3x + 10 3x − 6 + 16  16 
A= ∫ x − 2
dx A =
x−2
= ∫  3 + x − 2  dx
3 3 Sol 16:
4
= 3x + 16n ( x − 2 ) 
3

12 + 16  n2 − 9 − ln 1

= (3 + 16log 2) sq units.

π /2
Sol 13: ∫ y dx = ∫ x sinx2dx
0 x2 – 2ax + y2 ≤ 0

Substituting x2 = t y2 – ax ≥ 0
2 5 . 4 0 | Area Under the Cur ve and Linear Programming
2
y = 4x - x
a a a Sol 19: y = 4x – x2
A = ∫  2ax − x2 − ax  dx ⇒ A = ∫ 2ax − x2 dx − ∫ ax dx 4
   x3 
∫( )
0 0
0
4x − x2 dx = 2x2 − 
a  3 
2 3/2 0
a a2x 2
= π −
4 3 64 32
0 = 32 − = sq. units
3 3
πa2 2 a 3/2 πa2 2a2 a2
=
4

3
a =−
4 3
=
12
(3π − 8 ) sq. units
Exercise 2
Sol 17:
Single Correct Choice Type
4a
Sol 1: (C)
3 7

1
y = 4a ⇒ 16a2 = 4a2(x – 3) ⇒ x = 7
4a 4a  4a
y2   y3 
∫ x dy
== ∫  4a2 + 3  dy = 12a2 + 3y  1

∫ (e )
1
A= x ex + e− x 
− e− x dx =
0 0    0  0
0
( 4a)
3
64 1 1
= + 12a = a + 12a =e+ –1–1=e+ –2
12a 2 12 e e

 16 + 36  52a Sol 2: (A)


⇒  a =
 3  3 (1, 2)

52a 16a
A= − 3 × 4a = sq. units
3 3

y x2 + 3 and
Sol 18: The points of intersection of =
y 2x + 3 are (0, 3) and (2, 7).
=

Area = A1 + A2

1
1  x3/2 
 0 4
∫ 4x dx 2=
A1 = =
3/2 3
0
2
y −2
Equation normal = −1
x −1
⇒y–2=1–x
3
27 2
3
A1 = ∫ 2x + 3 = x + 3x = + 9 − 4 − 6 = 18 – 10 = 8 x+y=3 …… (i)
2
2 3
3 3
 x2  9 1
2
x3 8
2
A2 = ∫ ( 3 − x ) dx =3x −  = 9 – –3+
A2 = 2 2  2 2
∫x +3 =
3
+ 3x = + 6
3
1  1
0 0
=6–4=2
8 50
A1 + A2 = + 6 + 8 = sq. units 4 10
3 3 Area = 2 + =
3 3
M a them a ti cs | 25.41

Sol 3: (C) Sol 6: (B)



y=2

1 e

x=2

e 2 2
 2
A= ∫  lnx − (lnx )  dx ∫ ln=
dx x ln x − x Area = ∫ lnx dx + ∫ lny dy
1 1 1
2
A = (xlnx – x) – lnx ( xlnx − x ) − ∫ (lnx − 1) ==
2
2∫ lnx 2  xlnx − x  = 2[2ln2 – 2 + 1]
  1
1
= xlnx – x – x(lnx)2 + xlnx + xlnx – x – x
e
= 4ln2 – 2 = 2(2ln2 – 1)
 −x lnx 2 + 3xlnx − 3x 
 ( )  1
1
Sol 7: (B) y = x(1 – lnx) & x =
= –e + 3e – 3e + 3 = 3 – e e
1
Between x = &e
Sol 4: (A) e

2x

1
1 e
e
1 1 1

( )
3
= 2∫
A 2
x + 1 − 2x dx ∫ = 2 ⇒ (x − 1)
= 2 (x − 1) dx 2
=
2
0 0
3 3
0
x2 1  2 x2 
Sol 5: (A) (x – 1)2 + y2 = 1
I= ∫ ( x − xlnx ) dx = −  x lnx − 
2 2  2 
 πx  3x2 x2
If sin   I= − lnx
nx
 2  2 2
(1, 0) 2 e
 ⇒A=
I  1
e

3e2 e2  3 1  1 2 5
A= − − +  = e − 2
2 4 2  4e2 2e2  4 4e
 πx 
1 − ( x − 1 ) − sin
2
A= ∫  2 
 dx
0
Sol 8: (B)
2
 πx 
=  2x − x2 − sin 
 2 0
2
 sin−1 x − 1 + x − 1 2x − x2 
= 
( ) ( ) 2 πx
+ cos 
 2 π 2

π 2 π 2 π 4
 0
) )
2a
= + ( −1) + − = −
2×2 π 2.2 π 2 π
2 5 . 4 2 | Area Under the Cur ve and Linear Programming

π /2a x1
2
∫ ( cosax ) dx = a sinax  π /6a
π /2a
Area =
∫ f ( x ) = x1e 1 ⇒ f(x) = xe + ex
x x

π /6a 0

2 1 1  1
= 1 −  = a > 3 = a ∈  0,  Sol 12: (B)
a 2  3

x 1
Sol 9: (D) y = + ;y = 0
6 x2

2a 2a
x 1   x2 1 
A = ∫  +  dx =
 − 
a 
6 x2   12 x  a 0 0

2
a 1 a 1 a 1 2 2
Area = ∫ − −x − −x2 ( )= ∫− −x + x2
−1
= − − + = + −1
3 2a 12 a 4 2a  3 2 −x 3/2 
0

=  +
x ( )  = 1−2 = 1
a 1 3 3  3 3 3
Aleast when A’ = − =0⇒a=1   −1
2 2a2
x
Sol 13: (A) ∫ f(x)dx = 1 + 3x − 2
Sol 10: (C) 1

3
f(x) =
2 1 + 3x
O /2
3 3
x = 1, y = and x = 8, y =
4 10
8

 π/ 4 
∫ ydx 1 + 3.8 − 2 3
π/ 4
Area1 =  ∫ sinx  2 = 2  − cosx  A= 1
= =
0 ∆x 8 −1 7
 0 
dy
 1
= 2 −
 2

+ 1 =

( 2 −1 ) 2=
2− 2 Sol 14: (A) = 2x + 1
dx
y = x2 + x + c
π /2
π /2
(1, 2); c = 0
Area2 = ∫ cos x −=
area1 sinx 
0
−2+ 2
0

=1–2+ 2 = 2 –1

ratio is
2 ( 2 −1 )= 2 2 1
2 −1

y = x2 + x + 1
Sol 11: (D)
1 1
 x3 x 2  5
A = ∫ x + x=  +  =
2

0  3 2  0 6
M a them a ti cs | 25.43

Sol 15: (C) 3


For areamax = A’ = 3 – 2c = 0 ⇒ C =
2
 3 13 
 , 
2 4 
-3 -1 1 2
Sol 18: (C) A1 = a3
× a × 2a =
2
a
0 ≤ y < 3 – 2x – x2 a  x3  2a3
2
⇒ ==
∫ x = 
0 < y < 4 – (x + 1)2 −a  3  −a 3
1 2a3 4a3
2
= ∫ (3 − 2x − x )dx ⇒2 = 2a × a2 – =
3 3
0
A1 3
⇒ =
A2 4
Sol 16: (C)

Previous Years’ Questions

x2 y 2
Sol 1: (D) Given, + 1
=
1 9 5
O
C To find tangents at the end points of latus rectum, we
find ae.
1 i.e. ae = a2 − b2 = 4= 2
c

∫( )
2
x2 − cx3 dx =  4 5
0
3 and b2 (1 − e2 )= 5  1 − =
 9 3
1/c
 x3 cx 4  1 1 2 By symmetry, the quadrilateral is a rhombus.
⇒  −  = 3− 3 =
 3 4  3c 4c 3
0 y
A
1 2 1
⇒ = ⇒c=
12c3 3 2
2 2
L(ae, b (1-e ))
Sol 17: (C)
2 x’ O B x
(e, c +1) D

L’
O 1
C C

y’

y − c2 − 1 So, area is four times the area of the right angled triangle
= 2c
x−c formed by the tangent and axes in the 1st quadrant.
⇒ y = 2cx – c2 + 1  5
∴ Equation of tangent at  2,  is
x = 1, y = 2c – c2 + 1  3
x = 2, y = 4c – c2 + 1 2 5 y x y
x+ . =1⇒ + =1
1 9 3 5 9/2 3
⇒ Area of trapezoid = 6c − 2c2 + 2 × 1 = 3c – c2 + 1
2 
2 5 . 4 4 | Area Under the Cur ve and Linear Programming


x y ⇒ x = a (ax2)2
+ =1
9/2 3 1 1
⇒ x = 0, ⇒ y = 0,
∴ Area of quadrilateral ABCD a a
1 1
1 9  ∴ Point of intersection are (0, 0) and  , 
= 4 (area of ∆AOB)= 4  . .3  = 27 sq. units a a
2 2  Thus, required area OABCO = Area of curve OCBDO –
area of curve OABDO
Sol 2: (A) To find the area between the curves,
1/a  x 
=y + 3 x and x-axis in the 1st quadrant (we can
x ,2y = ⇒∫  − x2  dx =
1 (given)
plot the above condition as) ; 0  a 
 
1/a
 1 x3/2 ax3 
y’ ⇒ . −  1
=
 a 3 / 2 3 
0

2 1
B y= x ⇒ − 1
=
2
3a 3a2
1 1 ∴
x-3 ⇒ a2 = ⇒ d= ( a > 0)
y= 3 3
2
x’ x
O A(3,0) (9,0) Sol 4: (A) Since, tangents drawn from external point to
the circle subtends equal angle at the centre

y ∴ ∠O1BD = 30°
A
Area of shaded portion OABO
9 9 x − 3 
= ∫0 x dx − ∫   dx
3
 2  O3
9 9
 x3/2  1  x2 
=   −  − 3x 
3/2  
 0 2  2 3
O1 O3
2  1  81  9   1 cm 1 cm
=  .27  −  − 27  −  − 9  
3  2  2  2   B
30o 30o
C
1
= 18 – (18) = 9 sq. unit 3 cm D 2 cm E 3 cm
2

Sol 3: (A) As from the figure, area enclosed between O1D


In DO1BD, tan 30° =
the curves is OABCO. BD
Thus, the point of intersection of ⇒ BD = 3 cm

y = ax2 and x =2 ay2 2 2


Also, DE = O1O2 = 2 cm and EC = 3 cm
y = ax and x = ay
Y Now, BC = BD + DE + EC = 2 + 2 3
a
⇒ Area of ∆ABC
2
y=x
B
(1/a, 1/a) 3 3
C = = (BC)2 . 4(1 + 3)2
4 4
A
X’ O D
X = (6 + 4 3) sq. cm.

Sol 5: The area formed by |x| + |y| = 1 is square shown


as below,
Y’
M a them a ti cs | 25.45

y 1 1
and (x + 1)2 = ⇒–
4 2
-x + y = 1 x+y=1
1 1   1 1
x’
O
x ∴ Q  ,  and R  − , 
-1 1  2 4   2 4
x+y=1 x-y=1 ∴ Required area
1/2
y’ 1/2  1
2
 (x − 1)3 1 
= 2∫ (x − 1) −  dx = 2  − x
2
0
 4  3 4 
∴ Area of square = ( 2) = 2 sq. units 0

 1 1  1  8 1
Sol 6: Equation of tangent at the point (1, 3) to the = 2 − − −  − − 0  = = sq. unit
 8.3 8  3  24 3
curve
x2 + y2 = 4
Sol 8: (A) Region ( x, y ) : y 2 ≥ 2x
4,
is x + 3y =
x2 + y 2 < 4x and x ≥ 0, y ≥ 0
whose x-axis intercept (4, 0)
Y
y (2, 2)

A
P(1, 3)
O (2, 0) X
x’ x
(0, 0) O A(4, 0)

2
1
Area = π ( 2 ) − ∫ 2x dx
y’ 2

4 0
Thus, area of ∆ formed by (0, 0) (1, 3) and 2
 3 
(4, 0)  x2  2 2  2
3
= π− 2  = π− 2 
0 0 1  3  3  
 
1 1  
2 0
= 1 3=
1 | (0 − 4 3) | = 2 3 sq
sq.unit
units
2 2
4 0 1 2 2
= π− ×2 2
3

Sol 7: (A) The curves y = (x – 1)2, y = (x + 1)2 and y = 8


= π− sq. units
1/4 are shown are 3

Sol 9: (C) Region ( x, y ) : y < 2x and y ≥ 4x − 1


y y = (x + 1)
2 4
y = (x - 1) 2

y = 4x - 1
2
1/4 P y = 2x
R Q ( 1 ,1
2 (
y = 1/4
x
-1 -1/2 O 1/2 1
O

Where points of intersection are


( 18 , -12 (
1 1
(x – 1)2 = ⇒x=
4 2
2 5 . 4 6 | Area Under the Cur ve and Linear Programming

1  y + 1 y2  2m + 1 = n + 3 ⇒ 2m − n = 2
Area
= ∫  4 − 2  dy
−1/2   3m − 1 = 2n + k ⇒ 3m − 2n = k + 1
1
 y 2 y y3  1 1 1 1 1 1  = 9 4m + 1 =n ⇒ 4m − n =−1
=  + −  =  + − − + − 
3 4 6 32 8 48  32
 8 4 6  −1/2  On solving these equations, we get
3 9
Sol 10: (D) The point of intersection x − 3 =2 x m =− ,n =−5 ⇒ K =
2 2
⇒ ( x − 3) =
2
4x
⇒ x2 − 10x + 9 =0 Sol 13: (C) The required area
⇒x=
1,9
2 5 y  2
2  y 
⇒y=
1,3
= 2 ∫ 3 y−
  dy  = 2 ∫
 0 2
dy  = 5 ∫ y dy
3  0  2  
 
 0
3  3 
2y + 3 − y 2 dy =  y 2 + 3y − y 
Area ∫  3 
0  0 2
 y 3/2  10  3/2  10 × 2 2 20 2
=5   = = 2 =
= 9 + 9 − 9  = 9 sq. units 3   3 3
 3 / 2  0

Sol 11: (A)


Sol 14: (D)
2 2
x +y =1 y

y = sin x

5
4
x
O 
4

y = cos x

2
y =1-x π/ 4 5/ 4
Area = ∫ ( cos x − sinx )dx + ∫ ( sinx − cos x )dx
Area = Area of half circle 0 π/ 4
1 3π
 1 − x 3/2 
 ( ) 
1 1 2
1 − x dx = 2 π (1 ) + 2  − 3 / 2 
2
+2∫ + ∫ ( cos x − sinx )dx
0   0 5π
4
π/ 4
π 2 π 4 5π / 4
= + 2  = + = sinx + cos x  +  − cos x − sinx 
0 π/ 4
2 3 2 3
3π /2
+ sinx + cos x 
5π / 4
x −1 y +1 z −1
Sol 12: (C) = = = m
2 3 4 1 1  −1 1 1 1 
= + −1 +  − − − 
⇒ Any point on this line is ( 2m + 1,3m − 1, 4m + 1 ) 2 2  2 2 2 2
 1 1 
+  −1 + 0 + + 
x −3 y −k 2 2 2
Similarly for = = = n 
1 2 1
= 4 2 −2
(n + 3,2n + k,n)
It they intersect, then
M a them a ti cs | 25.47

= –2π – (π) = –3p


Sol 15: (B) y=x
⇒ A3 = (3π – 2) sq units.
xsinx = sinx ⇒ x = 1
⇒ a = 1 ⇒ A2 = π – 1 – sina
⇒ A1 = 1 – sina
y=1/x
t
1
2 × t sint2
O
x=1/e Sol 2: A = ∫ sinx dx , B = 2
x=1 0
t
2
1 1/e
1 A
∫ sinx dx
Area ∫ xdx + ∫ x
dx Now, Lim = 0
0 1 t →0 B 1
× t sint2
1 2
 x2  1/e 1
=   + Inx  = + In 1 / e 0 sint2
2
  0
1 2 form ⇒ Lim =
0 t 1
( 2t ) cos t2 + sint2
t →0

1 3 2 2
= +1 = sq. units
2 2 [L’ Hospital’s rule]

2sint2 2 tant2
= =
JEE Advanced/Boards 2t2 cos t2 + sint2 2t2 + tant2

Exercise 1 2 tant2
t2 2 2
a
= = =
tant 2 2+1 3
Sol 1: A1 = ∫ sinx − f(x) 2+
t2
0
a a
a
= – cos x − ∫ f(x) =
1 − cosa − ∫ f(x) Sol 3: f(x + 1) = f(x) + 2x + 1
0
0 0
f(0) = 1
π π π
π
⇒ A2= ∫ f(x) − sinx = ∫ fx + cos x= ∫ f(x) − 1 − cosa f(1) = 2⇒ f(x) = x2 + 1
a 2
a a a f(1) = 2 f(x) = x + 1
2π 2π 2π

⇒ A3 = ∫ sinx − f(x) = – cos x π
− ∫ f(x) =−2 − ∫ f(x) 2
π π π (t1t + 1)
⇒ A1 = 1 – sina + acosa – cosa
a
= 1 – cosa – ∫ f(x) dx
0
a
⇒ –acosa + sina = ∫ f(x) dx = sina – acosa
0
⇒ f(x) = xsinx f(–1) = 2;f(2) = 5;f(–2) = 5
π π y = x2 + 1
∫ f ( x ) dx =
π
∫ x sinx dx =
 −x cos x + sinx 
a
( )
Equation of tangent at P t,t2 + 1 y – (t2 + 1) = 2t(x – t)
a a

= π – [–acosa + sina] = π + acosa – sina Since, it passes through (0, 0)

⇒ A2 = π + acosa – sina – 1 – cosa –t2 – 1 = 2t(–t)

2π π ⇒ t2 = t ⇒ t = ±1

∫ ∫ x sinx dx =
f(x)dx =  −x cos x + sinx 
π ⇒ y = ±2x
π a
2 5 . 4 8 | Area Under the Cur ve and Linear Programming

∫ (x )
Area = 2 2
+ 1 − 2x dx ⇒ > 1 i. e. n = 2 +1
0
1
2 2 2
1 Sol 6:
= 2∫ ( x − 1 ) dx = ( x − 1 ) = 0 + 1 =sq. units
2 3

0
3 0
3 3

lnx − c
Sol 4: y = = 0 ⇒ x = ec 1
x

1
x   − (lnx − c)
x
f’(x) =   =0
x2
⇒ 1 – lnx + c = 0
a a
⇒ lnx = c + 1 1 1  ln(2x − 1) 
A= ∫ x − 2x − 1 = lnx −
 2

2
⇒ x = ec+1 2

ec +1 ec +1 ec +1
ln(2a − 1) ln3
lnx − c lnx c = lna – − ln2 +
dx b2 − 4ac 2 2
⇒ ∫ x
dx =∫
x
dx − ∫ x
ec c ec
3a 3a 4
e
= ln × 2 = ln − ln
2(2)(2a − 1) 4a − 2 5
ec +1
 (lnx)2  ec +1
=   − c lnx  ⇒ 15a = 16a – 8 ⇒a=8
ec
 2  ec
1
(c + 1)
2
− c2 2c + 1 1 Sol 7: f(x) =
= − c c + 1 − c  = − c  = 1 + x2
2 2 2  1  1 
We have, A(α, f(α)) and B  − ,f  −  
 α  α 
Sol 5: y = xn x
OA → y =  ……… (i)
α(1 + α2 )
y – 1 = n(x – 1)
−α3
1 1 OB → y = (x)
A
= n
∫ x dx − ∫ n(x − 1) + 1dx 1 + α2
α 0  
0 1  1 x  1 α3
1−
n ∫  1 + x2 − α(1 + α2 )  dx + ∫  1 + x2 + 1 + α2 x  dx
1
1 0  1 
 xn+1   x −1 2  −

=
  0 
− n
( ) + x
α
α 0
n+1  2   x2   α3 x 2 
 1− 1 = tan−1 x −  + tan−1 x + 
n  2α(1 + α2 )  0  2(1 + α2 )  − 1
α
1  n 1 1 1 α
= − 1 − −1 +  = − = tan–1α –
n+1  2n2 n  n + 1 2n
2(1 + α2 )
n−1   1 α 
⇒= + 0 + 0 − tan−1  −  −
2n(n + 1) 
  α  2(1 + α2 ) 
dA
= 0 ⇒2n(n + 1) – (n – 1) (4n + 2)=0
dn  1 (α + α )
= tan–1α – tan–1  − α  –
⇒ 2n2 + 2n – 4n2 – 2n + 4n + 2 = 0   2(1 + α2 )
⇒ –2n2 + 4n + 2 = 0 α2 − 1 (α + α )
= tan–1 1 − 1 −
⇒ n2 – 2n – 1 = 0 2(1 + α2 )

2± 8 π (α + α ) π 2α
n= = 1± 2 = − = −
2 2 2(1 + α ) 2 2(1 + α2 )
2
M a them a ti cs | 25.49

16 8  4
A’ = 0 ⇒ α = 1 = − + 2a=  2a − 
12 3  3
π 1
A= − 4 2
2 2 Is minimum ⇒ 2a = ⇒a=
3 3
−1
Sol 8: The curve is y = sin x, i.e., x = sin y. This is a
Sol 10: Sol.10
π π
standard curve. Lines x = 0, y = and y = − are the
2 2
π
y-axis and two lines parallel to the x-axis at a distance ,
2
one above and the other below the x-axis respectively.
Hence, the shaded part is the required area ∆. By
symmetry of the curve and the lines,
y – c = m(x – 1)
ar (OABO) = ar (OCDO)
∫ (x )
2
A= − m(x − 1) − c dx
∴∆= 2 × ar ( OABO )
b
π π  x3 m(x − 1)2 
2 2 =
 − − cx 
∫ ( x ) dy 2 ∫ sin y dy,
= 2=
curve
 3 2  a
0 0
∴  b3 − a3 m  
( b − 1 ) − ( a − 1 )  − c (b − a ) 
2 2
( The equation of the curve is x = sin y) =  −
π
 3 2   
∴ ∆= 2  − cos y  2= 2 0 + 1= 2
0
(b − a) (b2 + a2 + ab )
=
3
Sol 9: m 2
− b − a − 2b + 2a − c(b − a)
2
2 
c + mx – m = x2
⇒ x2 – mx + m – c = 0
a c b x1 + x2 = m
x1x2 = m – c

⇒ x1 – x2 = m2 − 4m + 4c
c  b

  ( )
A =  ∫ f(c) − f(x)  dx + ∫ f ( x ) − f ( c ) dx A  m2 − m + c m 
a  c ⇒ (b - a)  − m − 2  − c 
 3 2 
c b
= f(c)(c − a) − ∫ f(x) dx + ∫ f(x) dx − f(c)(b − c)  m2 − m + c m2 3c 3m 
a c ⇒ (b - a)  − − + 
c b  3 2 3 3 
A = f(c) [c – a – b + c] – ∫ f(x) dx + ∫ ( f(x)) dx  m2 2c 2m 
a c
⇒ (b – a)  − − + 
This is minimum when 2c – a – b = 0  6 3 3 
a+b 1 2
⇒c= A= + [m + 4c – 4m]3/2
2 6
2 3 2
x   x 4 x3 
∫ f(x) = ∫ 3
 − x 2
+ a  dx=  − + ax  m2 − 4m + 4c =
36

0   12 3  0
B2 − 4AC =
0
2 5 . 5 0 | Area Under the Cur ve and Linear Programming

(ii) For maximum Area of sq. ABCD


16 − 4 ( 4c − 36 ) =
0
Length 2 5 1 + c must be maximum
⇒ c = 10 ⇒ m = 2
For c = 63 (From Previous questions)
⇒ c2 + m2 = 104
( )
2
Area = 2 5 1 + c
Sol 11: A1 + A2 for any point P(t , t2)
(2 )
2
= 5 1 + 63 = 4 × 5 × 64
k t t

∫ C2dx − C + ∫ t dx − C= ∫ ( C − C1 ) dx
2
= 1280 sq. units.
0 k 0
(iii) Area bounded by y = 2x + 3 and
k k t 3
x  3  −x  3 3

∫ ( 2x + 3 − x )dx
t
2
∫ C2dx −  3  +  3  + t (t − k) = Area = 2

0  0  k 6 −1
3
k  x3 
k 3 k 3 t3 3 2 t3 =  x2 + 3x − 
∫ 2
C dx −
3
+ −
3 3
+ t − t k =
6  3 
−1
0
1
k
t3 2 = 9+ 9 − 9 −1 + 3 −
3
∫ C2dx =− 2 + t k
0 1 32
= 11 − = sq. units
Let C2 = f(x) l x 2 2 3 3
t/ λ
t3
∫( ) Sol 13:
λ2 x2 dx =+ t2k
0
2
2
t/ λ
a +1
λ  x3 
2
t3 t3 λ 2 t3 t3 t3
 0
=− + ⇒ =− +
3 2 λ 3 λ3 2 λ a

10 1 1 2 4
⇒ = − = ⇒ λ=
2 λ 3λ 3λ 3 Area of rectangle = a(a2 + 1)
a2 +1 a2 +1
16x2  2(y − 1)3/2 
⇒ c2 = A= ∫ y − 1 dy =
 
9 1  3 1
Sol 12: (i) Let equation of CD be y = 2x+c 2 3 2
= a − 0 = a3
3 3
For intersection with y = x2
2 3 1 2 4 3
⇒ x= 2x + c ; x − 2x − c =
2 2
0 == a a(a + 1) = a= a3 + a
3 2 3
2 and x1 x2 = −c
⇒ x1 + x2 =
a3 = 3a⇒ a = 3,– 3,0
Length of CD= x1 + x2 5 ⇒ a= 3
= 2 5 1+c x
2
c + 17
Sol 14: f3(x) = ∫ tf (t) dt
Length of AC
= BD
= 0
5
⇒ f’(x) 3f2(x) = xf2(x)
Given ABCD is square than,

c + 17 ⇒ f2(x)[3f’(x) – x] = 0
2 5 1+ c =
5 x x2
⇒ f’(x) = ⇒ f(x) = +c
⇒ c2 − 66 c + 189 =
0 3 6
⇒c=
3 ,63 x2
⇒ f(x) = +c
Therefore, least value of c is 3. 6
M a them a ti cs | 25.51

π /2 π
A= 2 ∫ sinx + 2 ∫ 2 − sinx
0 π /2
3 x 2
 x2   t2 
 + c =
 6 
∫  6 + c  dx
t  = 2  − cos x 
π /2
0
+ 2 2x + cos x 
π
π /2
0  

x6 3cx 4 3x2c2 = 2[+1] + 2[2π – 1 – p] = 2π = aπ + b


+ c3 + +
216 36 6 a = 2, b = 0

 t4 x
ct2 
x  t5 ct3  a2 + b2 = 4
2
2
= ∫ +c +  t dx = ∫  36
 + c t +  dx
3 
 36
0
3  0
Sol 17: –1 + x = 2y – y2 – 1
x
 t6 c2 t2 ct 4  x6 c2 x2 cx 4 x – 1 = –(y – 1)2
=  + +  = + + ⇒c=0
 216 2 12  216 2 12
0 y= 1 − x + 1 = mx + 2
2
x 1 – x = (mx + 1)2
⇒ f(x) =
6 1 – x = m2x2 + 1 + 2mx
3 3
 x3  3
Hence, ⇒ m2x2 + (2m + 1)x = 0
∫ f(x)dx
= = 
0   0 2
18
− ( 2m + 1 )
⇒ x1 = 0, x2 =
m2
Sol 15: Given curves are y = x + 1 2
…. (i)
− ( 2m + 1 )
and y =−x + 1 or y =− ( x − 1 )
2 2
…. (ii) x1x2 = 0, x1 + x2 =
m2
Curve (i) is the parabola having axis y = 0 and vertex
x2
(-1, 0).
A= ∫ (1 + 1 − x − mx − 2 dx )
Curve (ii) is the parabola having axis y = 0 and vertex x1
(1, 0) x
 (1 − x)3/2 mx2 2
(1) - (2) 2x = 0 x = 0 A = x − − − 2x 
 3/2 2  x
From (i), x = 0 ⇒ y =±1 1
x
 −2(1 − x) 1 − x mx2 2
Required area = area ACBDA =  − − x
 3 2  x
1 Y 1

∫ ( x1 − x2 ) dy
)
= C (0,1
2 m 
( )
3/2 
= − (1 − x2 ) − (1 − x1 )  −  x22 − x12  − ( x2 − x1 ) 
3/2

y=1
−1
3  2 
1

∫ (1 − y ) − (y )
− 1  dy
2 2
= 2 m
= − (1 − x ) − 1 − x2 − x
3/2
 A O B
−1
(-1, 0) (1, 0) 3  2
1
= 2 ∫ 1 − y 2 dy ( ) D(
0,-1
y = -1 2 2
= − (1 − x)3/2 −
mx2
−x
−1 ) 3 3 2
1
 y3   1   1  3/2
= 2  y − = 2 1 −  −  −1 +   2 2
= − 1 +
( 2m + 1)  –
2
m  2m + 1   2m + 1 
 3   3   3    + 
−1 3 3 m2  2  m2   m2 

8
= sq. units 2 2 2 1
3 = − (m + 1)3 + +
3 3m 3 m m2
y
Sol 16:
m 4 1 4  2 2 1 2 1
1 −  + + = + + − −
2  m2 m4 m3  3 m m2 m 2m3

O   3 2
2 2
2 5 . 5 2 | Area Under the Cur ve and Linear Programming

region R2 is the interior of the circle (ii). Therefore,


2 2 2 2 2 3 2 7 R1 ∩ R 2 is the shaded region.
− − − − − = − − −
m 2 3 3m m m
3 2
m2 m 6m3
Putting the value of y2 from (i) in (ii), we get
7 1 9
3m + 2m2 + 4x2 + 16x − 9 = 0 ⇒ x = −
A= 6 2 2
m3 1
From (i), x = ⇒ y =
± 2
2
Sol 18: 9
And x = − is not possible
2
1  1 
Thus, A =  , 2  and=
C  , − 2
2  2 

Required area = 2 area OABD


2

f’(x) = –xe–x + e–x = 2 ∫ ( x1 − x2 )dy


0
f”(x) = xe–x – e–x – e–x
2 2
= (x – 2)e–x = 0 1 y2
32 − ( 2y ) dy − 2
2
= 2 ∫2 ∫ 4
dy
⇒ x = 2 inflection point 0 0

2 2 2 2
2 1 1  y3 
−x  −xe− x − e− x 
∫ xe =  0
=
2 ∫
2 2
3 − z dz −
2
 
 3  0
0 0

= –2e – e + 1 = 1 – 3e
–2 –2 –2
[Putting z = 2y in first integral]

{( x, y ) : y ≤ 4x, 4x + 4y ≤ 9}
2 2
Sol 19: Let
= R 2 2 2
1  2 9 − z2 9 z 1
=  + sin−1  − .2 2
2  2 2 3 6
= {( x, y ) : y 2
≤ 4x} ∩ {( x, y ) : 4x + 4y ≤ 9} = R ∩ R
2 2
1 1
 0

1  2 2 2 2
Where
= R1 {( x, y ) : y 2
≤ 4x and
= R2 } {( x, y ) : 4 x 2
+ 4y 2 ≤ 9 } =+  2
2 
9
2
sin−1 −
3  6
Equation of the given curves are
2 9 2 2 2 9 1
y 2 = 4x = + sin−1 = + cos−1
… (i) 6 4 3 6 4 3
9
and 4x2 + 4y 2 = … (ii)
2 3 2 2
Y sin−1 = θ, then sin θ = and
3 3
2 2
4x + 4y = 9 A 1
2
y = 4x cos θ= 1 − sin2 θ=
3
y= 2
X 2 9 π 1
O
D B y=0 Required area = +  − sin−1 
6 4 2 3

 −1 π 
 cos x= − sin−1 x 
 2 
Curve (i) is a parabola having axis y = 0 and vertex (0, 0).  2 9π 9 1
=  + − sin−1  sq. units.
3  6 8 4 3 
Curve (ii) is a circle having centre at (0, 0) and radius . 
2
Clearly region R1 is the interior of the parabola (i) and
Sol 20:
M a them a ti cs | 25.53

Single Correct Choice Type

Sol 1: (D)
b b

∫( x2 − 1 − 3 + x dx) ∫ (x )
2
A= = + x − 4 dx
a a

b
 x3 x 2 
=  + − 4x 
y = a2x2 + ax + 1  3 2  a
2
 1 3 b3 − a3 b2 − a2
y =  ax +  + = + − 4(b − a)
 2  4 3 2

1 1 + 4 ( −1) 5 9 17 17
3
2
 1 = 17 + −4 = 17 − =
⇒A= ∫ 
 ax +  +  dx 3 2 3 2 6

0 
2 4

1
For point of intersection
 a2 x3 ax2  a2 a
⇒A=  + + x = − + + 1 x2 – 1 = 3 – x
 3 2  0 3 2
⇒ x2 + x – 4 = 0
2a 1
A least if A’=0 ie + =0 ⇒ b + a = –1 ⇒ ab = –4
3 2
⇒b–a= 1 + 16 ⇒ b – a = 17
3
a= −
4
Sol 2: (A) e–x and linearly = e–4 & x = 1
Sol 21:

∫ e − e−4  dx
−x
K a 
1   1 1
∫  x − nx  dx =∫  nx − x  dx = –e–4 – 4e–4 – (–e–1 – e–4)
1 K

1 = K  nK = –5e–4 + e–4 + e–1

KK = e 1 4 e3 − 4
= e–1 – 4e–4 = − =
K a e e4 e4
nx − xnx + x=
  xnx − x − nx 
1 K

 nK – 1 + K – 1 Sol 3: (D) y = 9 − x2 x2 – 6x + y2 = 0
= a  na – a –  na – 1 + K +  nK
(a – 1) = (a – 1) (  na)
(a – 1) [1 –  na] = 0
Either a = 1 or a = e
∴a=e

Exercise 2
2 5 . 5 4 | Area Under the Cur ve and Linear Programming

3/2
 9 − x2 − 6x − x2  dx
Area = 2 ∫  

0
3/2
 2 π/ 4 2π
 9 − x − 9 − ( x − 3)  dx = sin+ cos  + sin+ cos  +
2
= 2 ∫  
0 5π / 4
0
5π / 4
3/2
 − sin− cos 
π/ 4
 −1 x 2 
 9 sin 3 + x 9 − x 
= 2  – = 2 –1+1+ 2 – [– 2 – 2 ]= 4 2 sq. units
 2 
  0

 −1 x − 3 
3/2
Sol 6: (C)
 9 sin + (x − 3) 6x − x2 
2 3 
 2 
  0

π 3 9 π 3 9
= 9. + 9 − + 9. + 9−
6 2 4 6 2 4

π x 3 π 9 3  3 3
= 9. + 3×3 = 3 + = 3 π + 
3 2 6 2 2  f(x) = –1 + |x – 2|

g(x) = 2 – |x|
Sol 4: (B) f(x) =1 – x ∈ (0, 2)
x – 3 ∈ (2, 4)
g(x) =2 – x ∈ (0, 3)
2 4
2 + x ∈ (–1, 0)
g(f(x)) =2 – f (x) f (x) ∈ (0, 3)
2 3
 3  3
Area = ∫  x −  dx + ∫  4 − x −  dx =2 + f(x)f(x) ∈ (–1, 0)
x x
3 2
=2 – (1–x), x ∈ (0, 1) = 1 + x
2 3
 x2   x2 
=  − 3nx  +  4x − − 3nx  =2 + (1–x), x ∈ (1, 2) = 3 – x
 2  3  2  2
=2 + (x–3), x ∈ (2, 3) = x – 1
3 9 =2 – (x–3), x ∈ (3, 4) = 5 – x
= 2 – 3  n2 – + 3  n 3 + 12 – – 3  n3 – 8
2 2
+ 2 + 3  n2
1 15 1 3 4 − 3n3
= + − 6 + 3n = 2 − n3 =
2 2 3 2 2
1 2 3 4

Sol 5: (D)
3 9
Area = 1 + 2  × 1 =
2 2
O

Sol 7: (C)
sin x

Area = 
π/ 4 5π / 4

∫ ( cos x − sin x ) dx + ∫ ( sinx − cos x ) dx /2


x=0
0 π/ 4

+ ∫ ( cos x − sinx ) dx
5π / 4
M a them a ti cs | 25.55

π/ 4 1
Area = 2 ∫ ( sec x − 1) dx 1  2x3/2 
 =
2
0
Area1 = ∫ x dx = 
 3  0 3
0
π/ 4 π
= 2 n ( sec x + tanx )  − 1
0 2  2 − x2  dx
Area2 = ∫  

= 2ln ( 2 +1 −
π
2
)= ln 3 + 2 2 − (
π
2
) 0
1
 −1 x 
 sin + x 2 − x2 
Sol 8: (A) = 2 2  = π +1
a=1  2  4
 
 0
(3, 2)
π 2  π 1
Area = +1− =  + 
4 3  4 3

Sol 11: (D)

Eq. of tangent
y − 2 2×3 3
= = ⇒ 2y – 4 = 3x – 9
x −3 2 4 2
3x – 2y = 5  ... (i)
1 1
1
5
  3x − 5   πx 2 πx  2 2
A = × 2×1 + ∫ x −    dx A1 = ∫ sin dx = − cos  = − ( −1 ) =
2  2  2 π 2 0 π π
1 0

5 1
 −x2 + 10x  1 1
= 1+  = 1 + (16) = 5 A2 = ∫x
2
dx =
4 4 3
 1 0
1
3 6−π
Sol 9: (A) y = 1 + 4x – x2 Ratio = =
2 1 π

⇒ y – 5 = –(x – 2)2 π 3
3/2
2
Area = ∫ 1 + 4x − x Sol 12: (D) A = 
∫  y
2
− 1− | y | 1 − y 2  dy
0 
3/2
 x3  3 9 27 9 39 1 0
2
=  x + 2x −  = = 6− =  2 2   2 2 

 3  2
+ 2× −
4 24 8 8
= ∫  y − 1 − y 1 − y 3/2
 dy1+ ∫  y − 1 + y 1 − y  dy
  3/2 0
0 0  −1 
 2   2 
 3
y  1−y    3
y  1−y  
1 3 3m 1 39
Area of ∆ = × × = × =  −y+   + −y−  
2 2 2 2 8 3 3  3 3 
 2    
39 13  2 0   −1
9m = ⇒m=
2 6
1  1  1  1 
= −1 − 0 − 0 +  + 0 − 0 − −  − + 1
Sol 10: (A) 3  3  3  3 
1 1
=–1+ − − 1 = –2; Area = 2
3 3
2 5 . 5 6 | Area Under the Cur ve and Linear Programming

Sol 13: (B) f(x) = 3x3 + 2x 1 1


3 3 1
⇒ − {(1 − b) − 1} + {0 − (1 − b) } =
g(x) = f–1(x) 3 3 4
5 1 1
 3x 4  3  7 2 1 1 1
∫=g(x) ∫=f(x)  + x2  =  + 1 = ⇒ – (1 − b)3 =− + =−
0 0  4  0  4  4 3 3 4 12
1 1 1
⇒ (1 − b)3 =⇒ (1 − b) = ⇒ b =
8 2 2
Previous Years’ Questions 2
Sol 3: (C) R1 = ∫−1 x f(x)dx  ... (i)
π/ 4  1 + sinx 1 − sinx 
Sol 1: (B) Required area = ∫
 −  dx Using,
b b
0 
 cos x cos x  ∫a f(x)dx
= ∫ f(a + b − x)dx
a
 1 + sinx 1 − sinx 
=  > > 0 2
 cos x cos x 
R1 = ∫−1 (1 − x)f(1 − x)dx,
  [given, f(x) = f(1 – x)]
x x
 2 tan 2 tan  2
2 2 ∴ R1 = ... (ii)
 1+
 2 x
1−
x


∫−1 (1 − x)f(x)dx 
π/ 4 
1 + tan 1 + tan2 
= ∫  2 2 Given, R2 is area bounded by
−  dx
2 x x
0
 1 − tan 1 − tan2 
  f(x), x = –1 and x = 2
2 2
 2 x 2 x
 2
 1 + tan 1 + tan  ∴ R2 = ∫−1 f(x)dx  ... (iii)
 2 2 
  Adding Eqs. (i) and (ii), we get
x x
π/ 4
 1 + tan 1 − tan 
2 − 2 2
= ∫ 
0  x x
 dx

2R1 = ∫−1 f(x)dx  ... (iv)
 1 − tan 2 1 + tan 
 2  ∴ From Eqs. (iii) and (iv), we get

x x x 2R1 = R2
1 + tan − 1 + tan π/ 4
2 tan
π/ 4 2 2 dx = 2
= ∫0 ∫0 x
dx
x 1 − tan 2 Sol 4: The point of intersection of the curves x2 = 4y
1 − tan2
2 2 and x = 4y – 2 could be sketched as, are x = –1 and
x = 2.
x 1 x
Substitute tan = t ⇒ sec2 dx = dt ∴ Required area
2 2 2
2
tan
π
4t dt 2 x + 2   x2   1  x2 x3 
= = ∫−1  4  −  4   dx =  + 2x − 
∫0 8
 

  4  2 3 
(1 + t2 ) 1 − t2 −1

π 1  8 1 1 
As tan= 2 −1 =  2 + 4 −  −  − 2 +  
8 4  3 2 3 

2 −1 4t dt y
So, ∫0 2 2
2
(1 + t ) 1 − t x =4y

Sol 2: (B) Here, area between 0 to b is R1 and b to 1


to R2.
b 2 1 1
∴ ∫0 (1 − x) dx − ∫ (1 − x)2 dx = x
b 4
1 2
b 1
 (1 − x)3   (1 − x)3  1
⇒  −  = y’
 −3   −3 
 0  b 4
M a them a ti cs | 25.57

1  10  −7   1 9 9 y
=  −    = . = sq unit.
4  3  6  4 2 8
y = cot x y = tan x

a 8  4 8 
Sol 5: Here, ∫2  1 + x2  dx =∫a  1 + x2  dx /2 -
x’ x
O /4
-/2 -/4
a 4
 8  8
⇒  x −  = x − 
 x 2  x a

y’
 8  8
⇒  a −  − (2 − 4) = (4 − 2) −  a − 
 a   a
 1   3 1 
=  log − 0  +  log − log 
y
 2    2 2 

3 1 3
P(1, 3) = log − 2log log
=
2 2 2
x’ x
(0,0) O A(4,0) 1 1
⇒ − log = loge 3 sq. units
2 2

y’ Sol 7: Given curves =


y 5 − x2 and y = |x – 1| could be
sketche as shown whose point of intersection are.
Thus, area of ∆ formed by (0, 0) (1, 3) and (4, 0) y

0 0 1
1 1
= 1 3 =
1 (0 − 4 3) = 2 3 sq unit
2 2 y=x-1
4 0 1 y = -x + 1
8 8
⇒ a− +2 = 2−a+
a a
x’ x
16 5-1 1 2 5
⇒ 2a − 0
=
a
y’
⇒ 2(a2 – 8) = 0
5 – x = (x – 1)
2 2

∴ a = ± 2 2 , (neglecting –ve sign) ⇒5 – x2 = x2 – 2x + 1


a = 2 2. ⇒ 2x2 – 2x – 4 = 0
⇒ x = 2, – 1
 π π
tanx, − ≤ x ≤ ∴ Required area
Sol 6: Given, y =  3 3
cot x, π ≤ x ≤ π 2 1 2
 =∫ 5 − x2 dx − ∫ ( −x + 1)dx − ∫ (x − 1)dx
6 2 −1 −1 1
1 2
which could be plotted as, y-axis. 2  2   2 
x 5  x   −  −x + x  −  x − x 
=  5 − x2 + sin−1  
π/ 4 π /3  2  −1  2 1
∴ Required area = ∫0 (tanx)dx + ∫
π/ 4
(cot x)dx  2 2  5   −1 

=  − log | cos x |


π/ 4
+ logsinx 
π /3  5 −1 2   5 −1  −1  
0 π/ 4 =  1 + sin  −  −1 + sin   
 2 5  2  5 
2 5 . 5 8 | Area Under the Cur ve and Linear Programming

y y = tanx
 1 1   1 
− − + 1 + + 1 − 2 − 2 − + 1
 2 2   2 

5  −1 2 1  1
=  sin + sin−1 − O
A
1
2 5 5 2 x’ x
B /4 /2-1

5 −1  2 1 1 4 1
= sin  1− + 1− −
2  5 5 5 5  2

5 1 5π 1
= sin−1 (1) − = − sq. units y’
2 2 4 2
π 
⇒ (2x – y) =  − 1 
Sol 8: Given curves are x2 + y2 = 4, x2 = – 2y  2 

Thus, the required area ∴ Required area is OABO


π/ 4
=∫ (tanx)dx − area of ∆ALM
0
2
y=x
π/ 4 1
= log | sec x | − BL . AL
2 2
x +y =4
0 2
O 2
x’ x 1π π−2
-2 - 2 2 = log 2 −  − .1
22 4 

 1
=  log 2 −  sq. unit
-2  4
2
y’ x = - 2y dy
Sol 10: y = x (x – 1)2 ⇒ = x . 2 (x – 1) + (x – 1)2
dx
2 0 2 −x2 2
= ∫− 4 − x2 dx − ∫− x dx − ∫0 dx 1 2 4
2 2
2 ∴ Maximum at x = 1/3 ⇒ ymax =  −  =
3 3 27
 x2 
0 2 Minimum at x = 1
2 x3
= 2∫0 4 − x2 dx −   −
 2  3 2
 − 2 0 y 2
y = x(x-1)
2
x 4 x 2 max
= 2  4 − x2 − sin−1  − 1 − 4
 2 2 2 0 3
27
5 1 x’ x
= (2 − π) − = − π sq. units O 1/3 1 min
3 3

dy
Sol 9: y = tan x ⇒ = sec2 x
dx y’
 dy 
∴   =2 = (x – 1) (2x + x – 1)
 dx x = π
4 = (x – 1) (3x – 1)
π 
Hence, equation of tangent at A  , 1  is + – +
4 
y −1 π 1/3 1
= 2 ⇒ y – 1 = 2x –
x−π/4 2
⇒ ymin = 0
M a them a ti cs | 25.59

Now, to find the area bounded by the curve y = x(x – y


1)2, the y-axis and line x = 2 y=
log x
ex
y y = ex log x
B
C
x’ x
2 O 1/e 1
4
27
x’ x
O 1 A
x=2 y’
1  log x 
∴ The required area = ∫1/e  ex
− ex log x  dx

y’ 1 1
1  (logx)2   x2  e2 − 5
=   − e  (2logx − 1) = sq. units
∴ Required area e  2   4 1/e 4e
1/e
2
= Area of square OABC – ∫ y dx
0
 4a2 4a 1 f( −1) 3a + 3a 
2
2
0
2 2
= 2 × 2 − ∫ x(x − 1) dx = 4 − ∫ x − 2x + x dx
0
( 3 2
) 
Sol 12: Given,  4b2

4b 1

f(1)

=

3b 2
+ 3b


 
 x 4 2x3 x2 
2  2  f(2)  3c2 + 3c 
4  −
=− +  =4 −  16 − 16 + y  4c 4c 1
    
 4 3 2   
0 4 3 2
⇒ 4a2 f(–1) + 4a f(1) + f(2) = 3a2 + 3a, … (i)
10
= sq. units 4b f(–1) + 4b f(1) + f(2) = 3b + 3b
2 2
... (ii)
3
and 4c2 f(–1) + 4cf(1) + f(2) = 3c2 + 3c … (iii)
Sol 11: Both the curves are defined for x > 0. Both are Where f(x) is quadratic expression given by,
positive when x > 1 and negative when 0 < x < 1
f(x) = ax2 + bc + c and (i), (ii) and (iii)
We know, lim (logx) → −∞ ⇒ 4x2 f(–1) + 4x f(1) + f(2) = 3x2 + 3x
x → 0+
logx or {4 f(–1) –3}x2 + {4 f(1) –3} x + f{2} = 0 … (iv)
Hence, lim → −∞ , This, y-axis is asymptote of
x→ 0 + ex As above equation has 3 roots a, b an c
second curve. ∴ above equation is identity in x.

And lim ex log x [(0) x ∞ form] i.e., Coefficients must be zero.


x → 0+
1 ⇒ f(–1) = 3/4, f(1) = 3/4, f(2) = 0 … (v)
e  ∴
elogx  ∞  x =0 f(x) = ax2 + bx + c
= lim  − form  = lim
x→ 0 1 / x  ∞
+  x → 0+  1 
− 2  ∴ a = –1/4 , b = 0 and c = 1, using Eq. (v)
 x 
4 − x2
(using L‘Hospital’s rule) Thus, f(x) = shown as,
4
Thus, the first curve starts from (0, 0) but does not y
include (0, 0).
(0, 1) V
Now, the given curves intersect, therefore
logx
ex logx = A
ex x’
(-2, 0) (2, 0)
x
i.e. (e2x2 – 1) log x = 0
2
1 B(2t, -t + 1)
⇒ x = 1, (since x > 0)
e
y
2 5 . 6 0 | Area Under the Cur ve and Linear Programming

Let A( −2,0),B = (2t, −t2 + 1) v y – 3 = (x + 1)2 + c(x + 1), which passes through (2, 0).

Since, AB subtends right angle at vertex V(0, 1) ⇒ –3 = (3)2 + 3c


⇒ c = –4
1 −t 2
⇒ . = −1 ⇒ t= 4 ∴ Required curve y = (x + 1)2 – 4(x + 1) + 3
2 2t
⇒ y = x2 – 2x
∴B(8, –15)  x3  2
2

∴ Required area = ∫ (x − 2x)dx = − x2  2


Equation of chord AB is 0  3 
 0
−(3x + 6)
y=
2 8 4
= − 4 = sq. units
∴ Required area 3 3

8  4 − x2 3x + 6  Sol 14:
= ∫−2  4 + 2  dx
{( x, y ) ∈ R }
  2
Region
= :y≥ x + 3 ,5y ≤ x + 9 ≤ 15
8
 x 3
3x  2
= x − + + 3x  Plotting all the curves
 12 4 
 −2

 128  2 
= 8 − + 48 + 24 −  −2 + + 3 − 6   6
(-3 , ) B(1, 2)
 3  3  (-4, 1) F
5
C
125
= sq. units
3 D E(-3, 0)
A

Sol 13: Here, slope of tangent


Area = Area ( ABFE ) − Area ( AEB )
dy (x + 1)2 + y − 3 dy (y − 3)
= ⇒ = (x + 1) + ,X
dx (x + 1) dx (x + 1) + Area (DEFC ) - Area (D E C )

Substitute x + 1 = X and y – 3 = Y 1 −3

dy dY
=
32
5 −∫3
− ( −x − 3 dx + ) 11
10 −∫4
− ( )
x + 3 dx =
3
2
sq. units
⇒ =
dx dX

dY Y dY 1 Sol 15: (A, B, D)


∴ = X+ ⇒ − Y=
X
dX X dX X y
S T
y=1
1
∫ − x dX − log X 1
=I.F e= e=
X
y y= 1 U W
e

y= 1 R 2

e y=e-x
2
Q
y = x - 2x
O V P
x

x’
2
x Form figure x = 1 / 2 x= 1 x=1
O 2
S > Area ( OP QR )
y’ 1 1
⇒ S > 1× ⇒S>
∴ Solution is, e e
1 1 Y
Y.
= ∫ X. X dX + c ⇒ = X+c S > Area (P VUW ) + Area ( OS T V )
X X
M a them a ti cs | 25.61

 1  1 1
< 1 −  +1×
 12  e 2

1 1  1 
< + 1 −  D is correct
12 e 2

Now, e− x ≤ e− x if x ∈ ( 0, 1 )
2

1 1
−x − x2  1
∫ e dx ≤ ∫ e dx ⇒  1 − e  ≤ S B is correct
0 0  

1 1 1  1
1− > 1 +  and S > 1 −
e 4 e e

1 1 
⇒S> 1 + 
4 e

(B) and (D) Correct

You might also like